Вы находитесь на странице: 1из 96

KINEMATICS

KINEMATICS
THEORY AND EXERCISE BOOKLET

CONTENTS

S.NO. TOPIC PAGE NO.

1. Rest and Motion ................................................................ 3 – 11

2. Motion Under Gravity .......................................................... 12 – 15

3. Graphs ............................................................................. 15 – 26

4. Two Dimensional Motion ...................................................... 26 – 36

5. Relative Motion .................................................................. 36 – 49

6. Questions for short Answer .................................................. 50 – 52

6. Exercise -I ........................................................................ 53 – 70

7. Exercise - II ...................................................................... 71 – 76

8. Exercise - III ..................................................................... 77 – 86

9. Exercise - IV ..................................................................... 87 – 89

10. Exercise - V .................................................................... 90 – 93

11. Answer key ..................................................................... 94 – 96

394 - Rajeev Gandhi Nagar Kota, Ph. No. 0744-2209671, 93141-87482, 93527-21564
1 IVRS No. 0744-2439051, 0744-2439052, 0744-2439053 www.motioniitjee.com, email-hr.motioniitjee@gmail.com

Downloaded from www.iitjeephysics4u.com


Page # 2 KINEMATICS

KINEMATICS

In this lesson we shall study the geometry of motion i.e., kinematics.

Kinematics is used to relate displacement, velocity, acceleration

and time without reference to the cause of motion. We shall discuss

about kinematics of the particle here. Use of the word 'particles'

does not mean that our study will be restricted to small corpuscles;

rather, it indicates that in this lesson the motion of bodies possibly

as large as men, cars, rockets, or aeroplanes will be considered

without regard to their shape and size.

The entire lesson is divided into two sections. In the first

section we shall study about the motion in a straight line. In the

second section motion of particle in a plane specially projectile

motion and the concept of the relative motion of one particle with

respect to another will be discussed.

IIT-JEE Syllabus :

Kinematics in one and two dimension (cartesion coordinates only), projectiles;

Relative Motion.

394,50 - Rajeev Gandhi Nagar Kota, Ph. No. : 93141-87482, 0744-2209671


IVRS No : 0744-2439051, 52, 53, www. motioniitjee.com , hr@motioniitjee.com

Downloaded from www.iitjeephysics4u.com


KINEMATICS Page # 3

1. REST AND MOTION :


* An object is said to be in motion wrt a frame of reference S1, when its location is changing with
time in same frame of reference S1.
* Rest and motion are relative terms.
* Absolute rest and absolute motion have no meaning.
Motion is broadly classified into 3 categories.
1. Rectilinear and translatory motion.
2. Circular and rotatory motion.
3. Oscillatory and vibratory motion.

1.1 Rectilinear or 1-D Motion


When a particle is moving along a straight line, then its motion is a rectilinear motion.
Parameters of rectilinear motion or translatory motion or plane motion :
(A) Time :
* It is a scalar quantity and its SI unit is second(s).
* At a particular instant of time, a physical object can be present at one location only.
* Time can never decrease.
y
(B) Position or location - It is defined with respect to A C
some reference point (origin) of given frame of reference.
 r1 B
Consider a particle which moves from location r1 (at time t1)
 r2
to location r2 (at time t2) as shown in the figure below,
following path ACB. x

(C) Distance :
The length of the actual path traversed by the particle is termed as its distance.
Distance = length of path ACB.
* Its SI unit is metre and it is a scalar quantity.
* It can never decrease with time.
(D) Displacement :
The change in position vector of the particle for a given time interval is known as its displacement.
→   
AB = r = r2 − r1
* Displacement is a vector quantity and its SI unit is metre.
* It can decrease with time.
For a moving particle in a given interval of time
* Displacement can be +ve, –ve or 0, but distance would be always +ve.
* Distance ≥ Magnitude of displacement.
* Distance is always equal to displacement only and only if particle is moving along a straight line
without any change in direction.
(E) Average speed and average velocity :
Average speed and average velocity are always defined for a time interval.
Total dis tan ce travelled ∆s
Average speed(vav ) = =
Time int erval ∆t
  
 Displacement ∆r r −r
Average velocity (vav ) = = = 2 1
Time int erval ∆t t2 − t1
* Average speed is a scalar quantity, while average velocity is a vector quantity. Both have the same
SI units, i.e., m/s.
For a moving particle in a given interval of time
* Average speed can be a many valued function but average velocity would be always a single-
valued function.
* Average velocity can be positive, negative or 0 but average speed would be always positive.

394,50 - Rajeev Gandhi Nagar Kota, Ph. No. : 93141-87482, 0744-2209671


IVRS No : 0744-2439051, 52, 53, www. motioniitjee.com , hr@motioniitjee.com

Downloaded from www.iitjeephysics4u.com


Page # 4 KINEMATICS

(F) Instantaneous speed and instantaneous velocity


Instantaneous speed is also defined exactly like average speed i.e. it is equal to the ratio of total
distance and time interval, but with one qualification that time interval is extremely (infinitesimally)
small. The instantaneous speed is the speed at a particular instant of time and may have entirly
different value than that of average speed. Mathematically.
∆s ds S
v = lim = ...(4)
∆s →0 ∆t dt
When ∆s is the distance travelled in time ∆t. B

Distance
As ∆t tends to zero, the ratio defining speed becomes
finite and equals to the first derivative of the distance.
The speed at the moment 't' is is called the instantaneous D ∆S
speed at time 't'. θ
On the distance - time plot, the speed is equal to the slope A C
∆t
of the tangent to the curve at the time instant 't'. Let A t
and B point on the plot corresponds to the time t and t + O t t + ∆t time
∆t during the motion. As ∆ t approaches zero, the chord AB
becomes the tangent AC at A. The slope of the tangent Instantaneous speed is equal to the slope
of the tangent at given instant.
equal ds/dt, which is equal to the intantaneous speed at
't'.
DC ds
v = tanθ = =
AC dt

(G) Instantaneous velocity :


Instantaneous velocity is defined exactly like speed. It is
equal to the ratio of total displacement and time interval,
but with one qualification that time interval is extremely
(infinitesimally) small. Thus, instantaneous velocity can S
be termed as the average velocity at a particular instant
Position/displacement

of time when ∆ t tend to zero and may have entirely


B
different value that of average velocity : Mathematically.
B'
∆r dr D ∆S
v = lim = θ
∆t →0 ∆t dt
A ∆t C
As ∆ t tends to zero, the ratio defining velocity becomes t
O
finite and equals to the first derivative of the position t t + ∆t time
vector. The velocity at the moment 't' is called the Instantaneous velocity is equal to the slope
instantaneous velocity or simply velocity at time 't'. of the tangent at given instant.

The magnitude of average velocity |vavg| and average speed vavg may not be equal, but magnitude of
instantaneous velocity |v| is always equal to instantaneous speed v.

Ex.1 In 1.0 sec a particle goes from point A to point B moving in a semicircle of radius 1.0 m. The
magnitude of average velocity is
(A) 3.14 m/sec (B) 2.0 m/sec (C) 1.0 m/sec (D) zero

Total displacement d A
Sol. Average velocity = = 1m
Total time t o
D = AO + OB 1m
= 1 + 1 = 2m B
t = 1 sec (given)
2
⇒ mg of v of = 2m/sec
1

394,50 - Rajeev Gandhi Nagar Kota, Ph. No. : 93141-87482, 0744-2209671


IVRS No : 0744-2439051, 52, 53, www. motioniitjee.com , hr@motioniitjee.com

Downloaded from www.iitjeephysics4u.com


KINEMATICS Page # 5

Ex.2 A particle moves along a semicircular path of radius R in time t with


constant speed. For the particle calculate
(i) distance travelled, R
A B
(ii) displacement,
(iii) average speed,
(iv) average velocity,
Sol. (i) Distance = length of path of particle = AB = πR
(ii) Displacement = minimum distance between initial and final point
= AB = 2R
total dis tan ce πR
(iii) Average speed, v = =
time t
2R
(iv) Average velocity =
t

Ex.3 A body travels the first half of the total distance with velocity v1 and the second half with
velocity v2. Calculate the average velocity :
Sol. Let total distance = 2x. Then
x x  v1 + v 2  2x 2v1v 2
total time taken = v + v = x  v v  ∴ Average speed = = v +v
1 2  1 2   v1 + v 2  1 2
x 
 v1v 2 
(G-1) When velocity is given as a function of t :
Ex.4 Velocity-time equation of a particle moving in a straight line is,
v = (10 + 2t + 3t2)
Find :
(a) displacement of particle from the origin of time t = 1 s, if it is given that displacement is 20 m at
time t = 0
(b) acceleration-time equation.
Sol. (a) The given equation can be written as,
ds
v= = (10 + 2t + 3t 2 )
dt
ds = (10 + 2t + 3t2) dt
s t

or ∫
20

ds = (10 + 2t + 3t 2 )dt
0
or s – 20 = [10t + t2 + t3]01
or s = 20 + 12 = 32 m
(b) Acceleration-time equation can be obtained by differentiating the given equation w.r.t. time.
Thus,
dv d
a= = (10 + 2t + 3 t 2 ) or a = 2 + 6t
dt dt
SPECIMEN PROBLEM :
(A) WHEN EQUATION OF DISPLACEMENT IS GIVEN AND SPEED TO BE FIND OUT

Ex.5 If displacement is depend on time such that


x = 2t –2 then find out average speed upt to 4 sec.
Total distance
Sol. Average speed =
Total time
for Total distance
at t = 0 it is at x = – 2
at t = 1 it is at 0 m
at t = 4 it is at 6 m.
Total distance = |– 2| + 0 + 6 = 8 m
Average speed = 8/4 = 2m/sec

394,50 - Rajeev Gandhi Nagar Kota, Ph. No. : 93141-87482, 0744-2209671


IVRS No : 0744-2439051, 52, 53, www. motioniitjee.com , hr@motioniitjee.com

Downloaded from www.iitjeephysics4u.com


Page # 6 KINEMATICS

(B) WHEN VELOCITY IS GIVEN AS A FUNCTION OF TIME AND DISTANCE TO BE FIND OUT
In this type of question first find out at what instant the velocity is zero. If this instant is come in our
time limit then distance can be calculated by breaking the integration in two part with modulas

Ex.6 If velocity is depend on time such that v = 4 – 2t. Find out distance travelled by particle from 1
to 3 sec.
Sol. Velocity is zero (4 – 2t = 0) at t = 2 sec
2 3
dx
So for distance
dt
= 4 – 2t ⇒ dx = ∫ (4 – 2t)dt + ∫ (4 – 2t)dt
2
1

dx = 1 + 1 = 2m

(G-2) When velocity is given as a function of x


dx dx
v = f(x) ⇒
dt
= f(x) ⇒ ∫ f(x) = ∫ dt
Ex.7 If velocity is given by following function V = x2. Then find out relation between x & t (assume x =
1 m at t = 0)
Sol. Relation between v & x is
v = x2
dx dx
we know that v = ⇒ = x2
dt dt
x t x
dx  1
∫x ∫ dt
dx =
= dt ⇒ 2 ⇒ –  = t
x2 1 0  x 1

1 1
⇒ – +1=t⇒x=
x 1 –t

(E) Average and instantaneous acceleration.


When the velocity of a moving object/particle changes with time, we can say that it is accelerated.
Average acceleration,
  
 v2 − v1 ∆v Change in velocity
(aav ) = = =
t2 − t1 ∆t Time int erval
Instantaneous acceleration,

  dv
(a) = lim aav = = Rate of change of velocity
∆t → 0 dt
Acceleration is a vector quantity whose direction is same as that of change in velocity vector. Its SI
unit is m/s2.
* When direction of acceleration and velocity are opposite to each other, then acceleration is termed
as retardation.
  
 dv d2r  dv
* a = dt = =v 
d t2 dr

(E-1) When acceleration is given as a function of x

Ex.8 if a = 2x ; initially particle is at x = 2m and is moving with 3 ms–1. Then find out v at x = 5 m.
Sol. Given a = 2x
v 5
vdv

dx
= 2x ⇒ ∫
3
vdv = ∫ 2xdx
2
2
v 9
⇒ – = 25 – 4 ⇒ v2 – 9 = 21 × 2 ⇒ v = 51 ms–1
2 2

394,50 - Rajeev Gandhi Nagar Kota, Ph. No. : 93141-87482, 0744-2209671


IVRS No : 0744-2439051, 52, 53, www. motioniitjee.com , hr@motioniitjee.com

Downloaded from www.iitjeephysics4u.com


KINEMATICS Page # 7

(E-2) When acceleration is given as function of velocity

Ex.9 If a is depend on v in a following way.


a = v and at t = 0 x = 1m, v = 1m/s.
(a) Find out its velocity at t = 2 sec.
(b) Find out its velocity at x = 3 m
Sol. Given a = v
v 2
dv

dv
dt
=v ⇒ ∫
1
v
= dt∫
0
⇒ ln v = 2 ⇒ v = e ms–1 2

(b) Given a = v
v 3


vdv
dx
=v ⇒ ∫
1

dv = dx
1
⇒ v–1=2 ⇒ v = 3 ms–1

(E-3) When acceleration is given as a function of t.

Ex.10 The acceleration of a particle which is depend on time is given by following function
a = 2t + 1
and at time t = 0, x = 1m and u i = 2m/s.
Then find out displacement of the particle at t = 3 sec.
dv dv
Sol. ∴ We know that a = ⇒ = 2t + 1 ⇒ dv = (2t + 1) dt
dt dt
vf t

∫ dv = ∫ (2t + 1)dt
0
2

v f – 2 = t2 + t ⇒ v f = t2 + t + 2
dx
Now v=
dt
xf t


dx
dt
= t2 + t + 2 ⇒ ∫
1

dx = ( t 2 + t + 2)
0

t3 t2
xf = + + 2t + 1
3 2
So, xf at t = 3 sec is
(3) 3 (3) 2
= + + 2(3) + 1 ⇒ 20.5 m
3 2
So, after t = 3 sec the position of the particle is 20.5m but the displacement of the particle is
= 20.5 – 1 = 19.5 m

(E-4) Constant Acceleration Format

Deduce the following equations for unifromly accelerated motion by using intergration technique.
1
(A) v = u + at (B) s = ut + at2
2
a
(C) v2 – u2 = 2as (D) snth = u + (2n – 1)
2

394,50 - Rajeev Gandhi Nagar Kota, Ph. No. : 93141-87482, 0744-2209671


IVRS No : 0744-2439051, 52, 53, www. motioniitjee.com , hr@motioniitjee.com

Downloaded from www.iitjeephysics4u.com


Page # 8 KINEMATICS

First equation of motion. Acceleration is defined as


dv
a=
dt
or dv = adt ...(1)
When time = 0, velocity = u (say)
When time = t, velocity = v (say)
Integrating equation (1) within the above limits of time and velocity, we get
v t
t
∫ dv = ∫ a dt
u 0
or [ v]uv = a ∫ dt = a[t]
t
0
0

or v – u = a(t – 0)
or v = u + at ...(2)
Second equation of motion. Velocity is defined as
ds
v=
dt
or ds = v dt = (u + at) dt ...(iii)
When time = 0, displacement travelled = 0
When time = t, displacement travelled = s (say).
Integrating equation (3) within the above limits of time and distance, we get
s t t t t
 t2 

0

ds = (u + at) dt = u
0

0
dt + a ∫
0
t dt or [s]0s = u[t]0t + a 
 2 0
 t2 
or s – 0 = u (t – 0) + a  2 − 0 
 
1 2
or at
s = ut + ...(4)
2
Third equation of motion. By the definitions of acceleration and velocity,
dv dv ds dv
a=
= × = ×v
dt ds dt ds
or ads = vdv ...(5)
When time = 0, velocity = u, displacement travelled = 0
When time = t, velocity = v, displacement travelled = s
(say)
Integrating equation (5) within the above limits of velocity and displacement, we get
s v s v v
 v2 

0
a ds = ∫
u
v dv or a ds = ∫
0

u
v dv or a[s]0s =  
 2 u

v2 u2
or a[s − 0] =
− or 2as = v2 – u2
2 2
or v2 – u2 = 2as ...(6)
Fourth equation of motion. By definition of velocity,
ds
v=
dt
or ds = vdt = (u + at) dt ...(7)
When time = (n – 1) second, displacement travelled
= sn – 1 (say).
When time = n second, displacement travelled = sn
(say)

394,50 - Rajeev Gandhi Nagar Kota, Ph. No. : 93141-87482, 0744-2209671


IVRS No : 0744-2439051, 52, 53, www. motioniitjee.com , hr@motioniitjee.com

Downloaded from www.iitjeephysics4u.com


KINEMATICS Page # 9

Integrating equation (7) within the above limits of time and distance, we get
sn n n n n
 t2 

sn −1
ds = ∫
n −1
(u + at)dt or [s]ssn = u
n −1 ∫ dt + a ∫t dt or sn − sn −1 = u[t]nn −1 + a  
 2 n −1
n −1 n −1
a a
= u[n –(n – 1)] + [n2 – (n – 1)2] = u + [n2 – (n2 – 2n + 1)]
2 2
a
snth = u +
(2n − 1) ...(8)
2
where snth = sn – sn –1 = displacement in nth second.
Ex.11 A car starts from rest and accelerates uniformly for 20 seconds to a velocity of 72 km h–1. It
then runs at constant velocity and finally brought to rest in 200 m with a constant retardation.
The total distance covered is 600 m. Find the acceleration, retardation and the total time taken.
Sol. (i) Motion with uniform acceleration
5
Here, u = 0 ; t1 = 20 sec ; v = 72 × = 20 ms–1
18
∴ v = u + at1
20 = 0 + a × 20 or a = 1 m s–2
Distance travelled by car in this time (20 sec),
1 2 1
S1 = ut + at = 0 + × 1 × (20)2 = 200 m
2 2
(ii) Motion with uniform velocity.
As given, total distance = 600 m
we have calculated S1 = 200 m (with uniform acc.)
and S2 = 200 m (with retardation)
∴ Net distance for which body moves with uniform velocity,
S = 600 – S1 – S2
= 600 – 200 – 200 = 200 m
dis tan ce 200
∴ Time taken, t = = = 10 sec.
uniform velocity 20
∴ Total time of journey, t = (20 + 10 + 20) sec
t = 50 sec
Total displacement 600
Average velocity = = = 12 m/s .
Total Time 50
(iii) Motion with uniform retardation.
For this motion, initial velocity, u = 20 m s–1 and final velocity v = 0 ; S2 = 200 m
Acceleration a' = ?
Using, v2 – u2 = 2 a' S2
(0)2 – (20)2 = 2(a′) × 200
a′ = – 1 ms–2
Let t′ = time for which the body comes to rest.
∴ v = u + a′ t ′
0 = 20 – 1t′
∴ t′ = 20 sec.
C. SPECIMEN PROBLEM 2
a=2m/s
Ex.12 Find out distance travelled by the block u=10 m/s
in 10 sec. for a given situation.
Sol. First find out it what instant velocity of block becomes zero.
v = u + at
given : u = 10 m/s, a = – 2m/s2
⇒ 0 = 10 – 2t ⇒ t = 5 sec

394,50 - Rajeev Gandhi Nagar Kota, Ph. No. : 93141-87482, 0744-2209671


IVRS No : 0744-2439051, 52, 53, www. motioniitjee.com , hr@motioniitjee.com

Downloaded from www.iitjeephysics4u.com


Page # 10 KINEMATICS

So we calculate distance for two time intervals.


For first 5 sec.
1 1
S1 = ut – at2 ⇒ S1 = (10)(5) – (2) (5)2 = 25 m
2 2
for Next 5 sec (blockis travelling towards the starting point)
1
S2 = ut + at2
2
u =0
1
S2 = × 2 × 25 = 25 m
2
S = S1 + S2 = 50 m

(D) Reaction time :


When a particular situation demands our immediate action, it takes some time before we really respond.
Reaction time is the time a person takes to observe, think and act. For example, if a person is driving
and suddenly a boy appears on the road, then the time elapse before he applies the breaks of the car
is the reaction time. Reaction time depends on complexity of the situation and on an individiual.
One can measure one's reaction time by a simple experiment. Take a rule and ask your friend to drop it
vertically through the gap between your thumb and forefinger. As soon as it is dropped, note the time
elapsed ts before you catch it and the distance d travelled by the ruler. (In a particular case, y was
found to be 21.0 cm. Estimate reaction time).
Sol. As the ruler drops under free fall so u = 0, and g = 9.8 ms–2. The distance travelled d and the reaction
time tr are related by
1 2 d
d= gtr = 4.9tr2 Or, tr =
2 4.9
Here d = 21.0 cm = 0.21 m
0.21
∴ tr = ≈ 0.2 s
4.9
Note : a v=0
Definition : Time taken by a driver to react for a situation u
Reaction Time of the driver
is ∆t = t1 – t0
Total distance covered by the car before stopping t1
t=t0 t2
= distance covered in uniform motion during to to t1 + distance (brakes applied)

u2
cover in deaccelerated motion during t1 to t2 = Total distance = u(∆t) +
2a
Ex.13 Assume that a car is able to stop with a retardation of 8 ms–2 and that a driver can react to an
emergency in 0.5 sec. Calculate the overall stopping distance of the car for a speed of 60 km–1
of the car.
5 50
Sol. Here, u = 60 km h–1 = 60 × = ms−1
18 3
50
Since the application of brakes takes 0.5 s, before this the car was moving with uniform speed of ms −1 .
3
∴ Distance covered in 0.5 sec, with a uniform speed is
50 25
S1 = u × t = × 0.5 = m = 8.33 m
3 3
Now car begins to move with a retardation of 8ms–2
∴ Distance covered before coming to rest,
2a S2 = v2 – u2
2
 50 
0−  2500
or v2 − u2  3  = 50 × 50 = = 17.36 m
S2 = = 144
2a −2 × 8 9×2×8
∴ Total (overall) distance = S1 + S2 = 8.33 + 17.36 ⇒ S = 25.69 m

394,50 - Rajeev Gandhi Nagar Kota, Ph. No. : 93141-87482, 0744-2209671


IVRS No : 0744-2439051, 52, 53, www. motioniitjee.com , hr@motioniitjee.com

Downloaded from www.iitjeephysics4u.com


KINEMATICS Page # 11

Ex.14 Two buses A and B are at positions 50 m and 100 m from the origin at time t = 0. They start
moving in the same direction simultaneously with uniform velocity of 10 ms–1 and 5 ms–1.
Determine the time and postion at which A overtakes B.
Sol. Here we use equation of motion for constant velocity in Cartesian form.
Given x1 (0) = 50 m, x2 (0) = 100 m,
v1 = 10 ms–1, v2 = 5 ms–1
The positions of the two buses at any instant t are
x1 (t) = x1 (0) + v1t = 50 + 10 t
x2 (t) = x2 (0) + v2t = 100 + 10 t
When A overtakes B,
x1 (t) = x2 (t)
50 + 10t = 100 + 5t or 5t = 50
t = 10 s
x1 (10) = x2 (10) = 150 m
Thus A overtakes B at a position of 150 m from the origin at time t = 10 s.

Ex.15 A bus starts from rest with constant acceleration of 5 ms–2. At the same time a car travelling
with a constant velocity of 50 ms–1 overtakes and passes the bus. (i) Find at what distance will
the bus overtake the car ? (ii) How fast will the bus be travelling then ?
Sol. (i) Suppose the bus overtakes the car after covering distance s.
When the two meet, time taken t is same.
1 2 1 2
For bus, s = ut + at = 0 + × 5 t
2 2
For car, s = 50 t
5 2
∴ t = 50 t or t = 20 s
2
Hence s = 50 t = 50 × 20 = 1000 m.
(ii) v2 = u2 + 2as = 0 + 2 × 5 × 1000 = 10,000 or v = 100 ms–1
SPECIMEN PROBLEM
(E) Maximum Separation :
u=0
a=4m/s2
Ex.16 40 m/s

What is the maximum separation between car and scooter ?


Sol. Initially seperation between car & scooter increases and then decreases.
Separation between them will be maximum at an instant at which velocity of the car is equal to the
velocity of scooter.
Velocity car = Velocity scooter = 40 m/s
from v = u + at
40 = 0 + 4t ⇒ t = 10 sec
The distance travelled by scooter in 10 sec.
is S1 = 10 × 40 = 400 m
The distance travelled by car in 10 sec. is
1 2 1
S2 = ut + at =0+ × 4 × (10)2 = 200 m
2 2
So maximum seperation = S1 – S2 = 400 – 200 = 200 m
* When they meet both will travel the same distance.
Questions 14, 15, 16 can be solved in a relatively easier way using the concept of relative motion
which is explained later
Now you can try Questions 1 to 13 in Ex. 1 and 1 to 6 in Exercise II

394,50 - Rajeev Gandhi Nagar Kota, Ph. No. : 93141-87482, 0744-2209671


IVRS No : 0744-2439051, 52, 53, www. motioniitjee.com , hr@motioniitjee.com

Downloaded from www.iitjeephysics4u.com


Page # 12 KINEMATICS

2. MOTION UNDER GRAVITY :

I FORMAT : (When a body is thrown vertically upward) B


It includes two types of motion
(i) Deaccelerated motion from A to B because the direction upward
of velocity and acceleration is opposite. So speed motion downward motion
decreases (accelerated motion)
(deaccelerated
(ii) Accelerated motion from B to C because the direction of motion)
velocity and acceleration is same (downward). So speed u
increases A C
(a) Time of flight :
It is the time taken by the particle to reach the ground. If the particle is thrown vertically upward with
initial velocity u then
ui = u
a = – g (take downward direction negative)
from equation
1 2
S = ut + at ⇒ Snet = 0 (when particle again reaches the ground)
2
t = T (time of flight)
1 2 2u
0 = uT – gT ⇒ T =
2 g
(b) Maximum Height :
from v2 = u2 + 2as
at maximum height v = 0, s = Hmax
u2
⇒ 0 = u2 –2 gHmax ⇒ Hmax =
2g
(c) Final velocity
from v = u + at
2u  2u 
v = vf a = – g t=T= ⇒ vf = u – g  
g  g
vf = – u
i.e. the body reaches the ground with the same speed with which it was thrown vertically upwards as
it thrown vertically upward.
(d) Time to reach half of the maximum height :-
ui = u a = – g
1 2
from S = ut + at Hmax
2
H 1 2 Hmax/2
= ut – gt
2 2
H = 2ut – gt2 ⇒ gt2 – 2ut + H = 0 u

u2
2
2u ± 4u – 4gH 2u ± 4u 2 – 4g ×  u2
⇒ t= ⇒t= 2g ∵ Hmax =
2g  2g
2g
u(2 ± 2 )
t= ...(1)
2g
Equation 1 gives two value of time which corresponds to
u(2 – 2 )
t1 = (from ground to Hmax/2 in upward motion)
2g
u(2 + 2 )
t2 = (from ground to Hmax/2 in downward motion)
2g

394,50 - Rajeev Gandhi Nagar Kota, Ph. No. : 93141-87482, 0744-2209671


IVRS No : 0744-2439051, 52, 53, www. motioniitjee.com , hr@motioniitjee.com

Downloaded from www.iitjeephysics4u.com


KINEMATICS Page # 13

(e) Time to reach any general height h


Let us assume that particle reaches from A to B in time t1 & from A to C is time t2.
1 2 1
So from S = ut + at ⇒ h = ut – gt 2
2 2
gt2 – 2ut + 2h = 0 B C

2u ± 4u 2 – 8gh
⇒ t= h
2g
u
u – u2 – 2gh u + u 2 – 2gh
So, t1 = , t2 = A
g g
⇒ t1 + t2 = T (Time of flight)

II Format (Free fall) :


A body released near the surface of the earth is accelerated downward under the influence of force of
gravity.
(a) Time of Flight : (0,0)
u=0
1 2
from equation S = ut + at
2
S = – H, u = 0, a = – g
t = T (Let assume) H

1 2 2H
⇒ – H = (0)T – gt ⇒ T =
2 g
(b) Final Velocity when body reaches the ground
from v2 – u2 = 2as
s=–H v = vf u = 0 a = – g
⇒ vf
2
– 0 = 2 (–g) (–H) ⇒ vf = 2gH

Ex.17 A ball is thrown vertically upwards with a velocity u from the ground. The ball allains a maximum
height Hmax. Then find out the time and displacement at which ball have half of the maximum
speed.
Sol. Maximum speed of the ball is u
At point B and C ball have speed u/2 but direction
is opposite so from
B C
v = u + at
u/2 u/2
Let t1 is the time taken by the ball from point A to B and t2 is
the time taken by the ball from A to C h
u
From A to B = u – gt1 ...(i)
2 A
u
From A to C – = u – gt 2 ....(2)
2
u 3u
from (i) t1 = , from (ii) t2 =
2g 2g
from equation v2 – u2 = 2as
2
 u 2
⇒ v = ± u/2, u = u, a = – g ⇒   – u = – 2gh
 2

 u 
2
3u 2 3
h=  = Hmax  ∴ h= Hmax
8g  2g  4

394,50 - Rajeev Gandhi Nagar Kota, Ph. No. : 93141-87482, 0744-2209671


IVRS No : 0744-2439051, 52, 53, www. motioniitjee.com , hr@motioniitjee.com

Downloaded from www.iitjeephysics4u.com


Page # 14 KINEMATICS

Ex.18 A ball thrown vertically upwards with a speed of 19.6 ms–1 from the top of a tower returns to
the earth in 6 s. Find the height of the tower.
Sol. Here u = 19.6 ms–1
g = –9.8 ms–2
Net displacement, s = – h
Negative sign is taken because displacement is in
the opposite direction of initial velocity.

Tower
1 2
As s = ut + gt h
2
1
∴ – h = 19.6 × 6 + × (–9.8) × 62
2
= 117.6 – 176.4 = –58.8
or h = 58.8 m

Ex.19 A ball is thrown vertically upwards with a velocity of 20 ms–1 from the top of a multistoreyed
building. The height of the point from where the ball is thrown is 25 m from the ground. (i) How
high will the ball rise and (ii) how long will it be before the ball hits the ground?
Sol. (i) Here u = +20 ms–1, g = –10 ms–2
At the highest point, v = 0
Suppose the ball rises to the height h from the point of projection.
As v2 – u2 = 2gs
∴ 02 – 202 = 2 × (–10) × h or h = + 20 m.
(ii) Net displacement, s = –25 m
Negative sign is taken because displacement is in the opposite direction of initial velocity.
1 2
As s = ut + gt
2

1
∴ –25 = 20t + × (–10) × t2
2
or 5t2 – 20t – 25 = 0 or t2 – 4t – 5 = 0
or (t+ 1) (t – 5) = 0
As t ≠ –1, so t = 5s.

Ex.20 A ball thrown up is caught by the thrower after 4s. How high did it go and with what velocity was it
thrown ? How far was it below the highest point 3 s after it was thrown?
Sol. As time of ascent = time of descent
∴ Time taken by the ball to reach the highest point = 2 s
For upward motion of the ball : u = ?, v = 0, t = 2s, g = – 9.8 ms–2
As v = u + gt
∴ 0 = u – 9.8 × 2
or u = 19.6 ms–1
Maximum height attained by the ball is given by
1 2 1
s = ut + gt = 19.6 × 2 + × (9.8) × 22 = 19.6 m.
2 2
Displacement of the ball in 3 s,
1
s = 19.6 × 3 + × (–9.8) × 32 = 58.8 – 44.1 = 14.7 m
2
Distance of the ball from the highest point 3 s after it was thrown
= 19.6 – 14.7 = 4.9 m.

394,50 - Rajeev Gandhi Nagar Kota, Ph. No. : 93141-87482, 0744-2209671


IVRS No : 0744-2439051, 52, 53, www. motioniitjee.com , hr@motioniitjee.com

Downloaded from www.iitjeephysics4u.com


KINEMATICS Page # 15

Ex.21 A balloon is ascending at the rate of 9.8 ms–1 at a height of 39.2 m above the ground when a
food packet is dropped from the balloon. After how much time and with what velocity does it reach
the ground?
Take g = 9.8 ms–2.
Sol. Initially the food packet attains the upward velocity of the balloon, so
u = 9.8 ms–1, g = 9.8 ms–2 , s = –39.2 m
Here s is taken negative because it is in the opposite direction of initial velocity.
1 2
Using, s = ut + gt , we get
2
1
– 39.2 = 9.8 t – × 9.8 t2
2
or 4.9 t2 – 9.8 t – 39.2 = 0 or t2 – 2t – 8 =0
or (t – 4) (t + 2) = 0 or t = 4 s or – 2 s
As time is never negative, so t = 4s.
Velocity with which the food packet reaches the ground is
v = u + gt = 9.8 – 9.8 × 4 = – 29.4 ms–1.
Negative sign shows that the velocity is directed vertically downwards.

When a particle is dropped then it will automatically attains the velocity of the frame at that time.
Ex.22 Two balls are thrown simultaneously, A vertically upwards with a speed of 20 ms–1 from the
ground, and B vertically downwards from a height of 40 m with the same speed and along the
same line of motion. At what points do the two balls collide? Take g = 9.8 ms–2.
Sol. Suppose the two balls meet at a height of x from the ground after time t s from the start.
For upward motion of balls A :
u = 20 ms–1, g = – 9.8 ms–2 u=20 ms–1 B
1 2
s = ut + gt
2 40–x

40 m
1
x = 20 t – × 9.8 t2 = 20t – 4.9 t2 ...(i) C
2
For downward motion of ball B, x
1
40 – x = 20 × t + × 9.8 t2
2 u=20 ms–1 A
= 20t + 4.9 t2 ... (ii)
Adding (i) and (ii), 40 = 40 t or t = 1 s
From (i), x = 20 × 1 – 4.9 × (1)2 = 15.1 m
Hence the two balls will collide after 1 s at a height of 15.1 m from the ground.

3. GRAPHS :

(i) Straight line :


A linear relation between y & x represents a straight line.
General equation of straight line
y = mx + c
m ≡ slope of line
c ≡ y intercept i.e. where the line cuts the y-axis. m=tanθ
Slope is defined as the tan of angle made by the m=tanθ
straight line with positive x-axis in anticlockwise θ θ
direction.
m < 0 ⇒ θ > 90°
m > 0 ⇒ θ < 90°
0° ≤ θ < 180°

394,50 - Rajeev Gandhi Nagar Kota, Ph. No. : 93141-87482, 0744-2209671


IVRS No : 0744-2439051, 52, 53, www. motioniitjee.com , hr@motioniitjee.com

Downloaded from www.iitjeephysics4u.com


Page # 16 KINEMATICS

Ex.23 Draw the graph for the equation : 2y = 3x + 2


3 (0,1) 3
Sol. 2y = 3x + 2 ⇒ y = x + 1 tanθ =
2 θ 2
3
m= > 0 ⇒ θ < 90°
2
c = +1 > 0
⇒ The line will pass through (0, 1)

Ex.24 Draw the graph for the equation : 2y + 4x + 2 = 0 tanθ = –2


Sol. 2y + 4x + 2 = 0 ⇒ y = – 2x – 1 θ
m = – 2 < 0 i.e., θ > 90°
c = – 1 i.e.,
line will pass through (0, –1) (0,–1)

: (i) If c = 0 line will pass through origin.


(0,c)

(ii) y = c will be a line parallel to x axis.


(c,0)
(iii) x = c will be a line perpendicular to y axis (0,0)
(0,0)
(ii) Parabola
A general quadratic equation represents a parabola.
y = ax2 + bx + c a≠ 0
if a > 0 ; It will be a opening upwards parabola.
if a < 0 ; It will be a opening downwards parabola.
if c = 0 ; It will pass through origin.

2
y=4x +3x

e.g. y = 4 x2 + 3x 2
y=–4x +3x

Average velocity & instantaneous velocity from Position vs time graph


Average velocity from t1 to t2

displacement x2 – x1 B
= = t –t x2
time taken 2 1
x2–x1
= tan θ = slope of the chord AB

x1 A θ
x 2 – x1 t2–t1
vinstantaneous = as lim
t2 → t1 t 2 – t1 t1 t2
when t2 approaches t1 point B approaches Point A and the chord AB becomes tangent to the curve.
Therefore
vinstantaneous = Slope of the tangent x – t curve

394,50 - Rajeev Gandhi Nagar Kota, Ph. No. : 93141-87482, 0744-2209671


IVRS No : 0744-2439051, 52, 53, www. motioniitjee.com , hr@motioniitjee.com

Downloaded from www.iitjeephysics4u.com


KINEMATICS Page # 17

(iii) Reading of Graph


(A) Reading x v/s t graphs Explanation
x

x0
(1) Body is at rest at x0.

t
x

(2) Body starts from origin and is moving with speed tan θ away from origin.

θ t
x

(3) Body starts from rest from origin and moves away from origin with increasing
speed velocity and positive acceleration.

x
(4) Body starts from rest from x = x0 and moves away from origin with increasing
velocity or +ve acceleration.
x0

(5) x0 Body starts from x = x0 and is moving toward the origin with constant velocity
passes throw origin after same time and continues to move away from origin.

x
x0
(6) Body starts from rest at x = x0 and then moves with increasing speed towards
origin
∴ acceleration is –ve
t

x
(7) Body starts moving away from origin with some initial speed. Speed of body is
decreasing till t1 and it becomes 0 momentarily of t = t1 and At this instant. Its
reverses its direction and move towards the origin with increasing speed.

t2
t
O t1

394,50 - Rajeev Gandhi Nagar Kota, Ph. No. : 93141-87482, 0744-2209671


IVRS No : 0744-2439051, 52, 53, www. motioniitjee.com , hr@motioniitjee.com

Downloaded from www.iitjeephysics4u.com


Page # 18 KINEMATICS

(8) x Body starts from origin moves away from origin in the –ve x-axis at t = t1 with
decreasing speed and at t= t1 it comes at rest momentarily, Reverses its direction
t1 moves towards the origin the increasing speed. Crosses the origin at t = t2.
t
t2

x
(9) Body starts from origin from rest and moves away from origin with increasing
t speed.

(B) V-T GRAPHS


v

(1) Body is always at rest.

t
v

(2) v0 Body is moving with constant velocity v0

t
v

(3) Body is at rest initially then it starts moving with its velocity increasing at a
constant rate i.e. body is moving with constant acceleration.

v t

(4) Body starts its motion with initial velocity v0 and continues to move with its
v0 velocity increasing at a constant rate i.e. acceleration of the body is constant.

t
v
(5) Body starts its motion with initial velocity v0. Then it continues to move with its
velocity decreasing at a constant rate i.e. acceleration of the body is negative
v0
and constant. At t = t0 the body comes to rest instantaneously and reverses its
t0 direction of motion and then continues to move with decreasing velocity or increasing
t speed.
For 0 < t < t0 motion of the body is deaccelerated (∴ speed is decreassing)
t > t0 motion of the body is accelerated (∴ speed is increasing)
v

(6) Body is at rest initially. Then it starts moving with increasing velocity. As time
increases its velocity is increasing more rapidly. i.e. the moving with increasing
acceleration.
t

394,50 - Rajeev Gandhi Nagar Kota, Ph. No. : 93141-87482, 0744-2209671


IVRS No : 0744-2439051, 52, 53, www. motioniitjee.com , hr@motioniitjee.com

Downloaded from www.iitjeephysics4u.com


KINEMATICS Page # 19

v
(7) v0 Body starts its motion with initial velocity v0. Its velocity is decreasing with
time and at t = t0 . It becomes zero after body reverse its direction of motion and
continues to move with decreasing velocity or increasing speed. Since velocity
of the body is decreasing for whole motion. Therefore, its acceleration is
t0 t negative.For 0 < t < t0 motion of the body is deaccelerated (speed is
decreassing) t > t0 motion of the body is accelerated (∵ speed is increasing)

(C) READING OF a - t GRAPHS

(1) acceleration of the body is zero that means the body is moving constant velocity.

(2) Acceleration of the body is constant and positive.


t

t
(3) Acceleration of the body is constant and negative

(4) Initially the acceleration of the body is zero. Then its acceleration is increasing
at a constant rate.
t

(5) The body starts accelerating(initial acceleration zero) at t = 0. Its acceleration is


t
negative for whole of its motion and is decreasing at a constant rate.

(6) Initially acceleration of the body is zero. Its acceleration is positive for whole of
its motion. Its acceleration is increasing for whole of its motion.
t

394,50 - Rajeev Gandhi Nagar Kota, Ph. No. : 93141-87482, 0744-2209671


IVRS No : 0744-2439051, 52, 53, www. motioniitjee.com , hr@motioniitjee.com

Downloaded from www.iitjeephysics4u.com


Page # 20 KINEMATICS

(IV) Drawing of graphs on the basis of given information.


(a) If acceleration of the body is zero.
(i) If the velocity of the body is v0 and it starts from origin.
x v
t
v0
0
u
x=

t t
(ii) If at t = 0, x = x0 then
x v
t
+v
0

x0 x 0
v0
x=

t t
(iii) If at t = 0, x = – x0 then
x v
t t
+v
0
v0
–x
0

x=
t t
–x0

(b) If a body has constant acceleration :


For this section
(i) u0, x0 & a0 are positive constants. (ii) u ≡ initial velocity (iii) v ≡ velocity at any time t.
(iv) x ≡ Position at any time t.
xi ≡ initial position
(i) if u = 0, a = a0
1
if xi = 0, x = at 2 if xi = x0, x = x0 + (1/2)at2
2 x
x x
This is wrong because
it suggest the body don't
have some initial velocity
x0
t
t t
v a

a0
slope = tanθ
= a0
θ
t t
v = a0 t
(ii) If u = u0 , a = a0
1
x = xi + u0t + a0 t 2 v = u0 + a0t
2
x x
v a

a0

if xi = 0 t if xi = x0 t t t

394,50 - Rajeev Gandhi Nagar Kota, Ph. No. : 93141-87482, 0744-2209671


IVRS No : 0744-2439051, 52, 53, www. motioniitjee.com , hr@motioniitjee.com

Downloaded from www.iitjeephysics4u.com


KINEMATICS Page # 21

(iii) if u = u0, a = – a0
1 2
x = xi + u0t – a 0 t
2
x
x

x0
if xi = 0 if xi = x0
t t
t0

v
a
u0
t
t0
t –a0

(iv) if u = – u0 , a = + a0
1
x = xi – u0t + a0 t 2
2
x

x0

if xi = 0 if xi = x0
v
a

a0
t
–u0 t
(v) If u = u0, a = – a0
1
x = xi – u0t – a0 t 2
2
x x
x0

t t

if xi = 0 if xi = x0
v
a

t t
–v0
–a0

394,50 - Rajeev Gandhi Nagar Kota, Ph. No. : 93141-87482, 0744-2209671


IVRS No : 0744-2439051, 52, 53, www. motioniitjee.com , hr@motioniitjee.com

Downloaded from www.iitjeephysics4u.com


Page # 22 KINEMATICS

Ex.25 Draw the


(a) position vs time graph
(b) velocity vs time graph
(c) acceleration vs time graph
for the following cases
(i) If a body is projected vertically upwards with initial velocity u. Take the projection point to be
origin and upward direction as positive.
1 2
x = ut – gt
2
x v
a
2
u u
u 2u
2g
g g
t t
t
u 2u
g g –u –g

(ii) If a body is dropped from a height h above the ground. Take dropping point to be origin and
upward direction as +ve.
1 2
x= – gt
2
x v a=–g
a
2h 2h
g g
t t
t
v = – gt

–h – 2gh –g

(iii) If a body is projected vertically upwards from a tower of height h with initial velocity u. Take
the projection point to be origin and upward direction as +ve.
x
u2 v
2g a
2u
g
t u/2g t
u v = u – gt t
g
–g
–h

(iv) A car starting from rest accelerates uniformly at 2 ms–2 for 5 seconds and then moves with
constant speed acquired for the next 5 seconds and then comes to rest retarding at 2 ms–2.
Draw its
(a) Position vs time graph
(b) Velocity vs time graph
(c) acceleration vs time graph

394,50 - Rajeev Gandhi Nagar Kota, Ph. No. : 93141-87482, 0744-2209671


IVRS No : 0744-2439051, 52, 53, www. motioniitjee.com , hr@motioniitjee.com

Downloaded from www.iitjeephysics4u.com


KINEMATICS Page # 23

acceleration vs time graph


x (in m)
x
100
a
+2 v
75
10 15 –1
t 10ms
5
15 25
–2 t
5 10
t (in sec)
5 10 15
acceleration vs time graph velocity vs time graph Position vs time graph

(v) A particle starts from x = 0 and initial speed 10 ms–1 and moves with constant speed 10ms–1 for
20 sec. and then retarding uniformly comes to rest in next 10 seconds.
acceleration vs time graph
a
v

–1
10ms
20 30
t (sec)

–2 t (sec)
–1ms 20 30
Acceleration vs time graph velocity vs time graph

x
250m

200m

t (sec)
20 30sec

Position vs time graph

(V) Conversion of velocity v/s time graph to speed v/s time graph.
As we know that magnitude of velocity represent speed therefore whenever velocity goes –ve take its
mirror image about time axis.

velocity
speed
e
ag
i m
r
i rro
Ex-26 m
t (sec)
t (sec)

velocity speed e
ag
im
or
irr
Ex-27 m
t t

394,50 - Rajeev Gandhi Nagar Kota, Ph. No. : 93141-87482, 0744-2209671


IVRS No : 0744-2439051, 52, 53, www. motioniitjee.com , hr@motioniitjee.com

Downloaded from www.iitjeephysics4u.com


Page # 24 KINEMATICS

(VI) Conversion of displacement vs time graph to distance vs time graph


D

Dist./Displacement
For distance time graph just make the mirror image of the displacement
Dist.-time
time graph from point of zero velocity onwards. C

(VII) Conversion of v - t graphs in to x-t and a-t graphs B Disp-time


v x A
Time

v0

t0
(i) ⇒ v
x=

t t
v x a

a0
0
a
=

(ii) ⇒ a - t graph
ta

t t t

v x
a
tan θ = – a0
t
(iii) ⇒
θ
t0 t
t –a0
t0
at t = t0 velocity reverses its direction.

v
(iv) x – t graph
From t = 0 to t = t1 acceleration = 0 therefore v0
from t = 0 to t = t1, x - t graph will be a straight line.
From t = t1 to t2 acceleration is negative
∴ It will be an opening downward parabola
x t1 t2 t

t1 t2 t
v
(v) upto t = t1 acceleration is +ve
t1 < t < t2 acceleration is zero.
t > t2 acceleration is –ve
x t
t1 t2

x - t graph
t1 t2 t

394,50 - Rajeev Gandhi Nagar Kota, Ph. No. : 93141-87482, 0744-2209671


IVRS No : 0744-2439051, 52, 53, www. motioniitjee.com , hr@motioniitjee.com

Downloaded from www.iitjeephysics4u.com


KINEMATICS Page # 25

Some important points :


dv
• a=
dt ∫
⇒ dv = adt ∫
⇒ ∆v = area under the a - t curve
dx
• v=
dt ∫ ∫
⇒ dx = vdt
⇒ ∆x = area under the v - t curve
⇒ displacement = area under the v - t curve
a
Ex-28 If at t = 0 u = 5 ms–1 then velocity at t = 10 sec
= u + change in velocity –2
5ms
= 5 + area of the shaded part
= 5 + 10 × 5
= 55 ms–1 10 sec t
a
Ex-29 if at t = 0, u = 2 ms–2 find out it maximum velocity 5ms–2
Since whole motion is accelerating. Therefore velocity
will be max at the end of the motion which will be
1
=2+ × 5 × 10 = 27 ms–1 t
2 10sec
Ex-30 if at t = 0, u = 4 ms–1 a
Find out v at
t = 10 sec, t = 20 sec & t = 30 sec. –2
10ms
Since for whole motion acceleration of the body is positive
1
vt= 10 sec = 4 + × 10 × 10 = 54 ms–1
2 t
1 10sec 20sec 30sec
vt = 20 sec = 4 + × 10 × 10 + 10 × 10
2
= 154 ms –1

1
vt = 30 sec = 154 + × 10 × 10 = 204 ms–1
2
(VIII) Reading of graphs if the motion of two bodies are sketched on the same axes.
(a) Reading of x - t graphs
x

x3
B

x2

x1
A

O t1 t2 t3 t
Conclusions :
(i) Body A Start its motion at t = 0 from origin and is moving away from the origin with constant velocity.
Finally it ends its motion at a distance of x2m from origin at t = t3.
(ii) Body B starts its motion at t = t1 from origin and is moving away from origin with constant velocity.
Finally it ends its motion at a distance of x3m from origin at t = t3
(iii) Since slope of B is greater than slope of A. Therefore velocity of B is greater than velocity of A.
(iv) A t = t2, Both A & B are at the same distance from starting point that means B overtakes A at t = t2
(v) ∵ velocity of both A & B are constant
∵ acceleration of both the bodies are zero.
(vi) ∴ x3 > x2
∴ At the end of the motion B is at a greater distance from the starting point.

394,50 - Rajeev Gandhi Nagar Kota, Ph. No. : 93141-87482, 0744-2209671


IVRS No : 0744-2439051, 52, 53, www. motioniitjee.com , hr@motioniitjee.com

Downloaded from www.iitjeephysics4u.com


Page # 26 KINEMATICS

x2 A

B
x1
Ex-31 x0

t0 t1 t
Conclusion :
(i) Body A starts its motion at t = 0 from origin and is moving away from the origin with constant
velocity. Finally its motion ends at t = t1 at x = x2 m.
(ii) Body B starts its motion at t = 0 from x = x0 and then moves with constant velocity away from the
origin. Finally it ends its motion at t = t1.
(iii) Velocity of A is greater than that of B.
(iv) At t = t0 A overtakes B
(v) acceleration of both A & B is zero.
(vi) ∵ x2 > x1
∴ At the end of the motion A is at a greater distance from the starting point then B

x
B
A

Ex-32

t1 t2 t
Conclusions :
(i) Both A & B starts their motion at same time t = 0 and from same point x = 0.
(ii) Both are moving away from the starting point.
(iii) A is moving with constant velocity while B starts its motion from rest and its velocity is increasing
with time i.e. it has some positive acceleration.
(iv) ∵ At t = t1 the tangent on B's graph becomes parallel to the A's graphs
∴ At t = t1 velocity of both A & B is same.
(v) For t < t1 velocity of A is greater than velocity of B. Therefore up to t = t1, separation between A
& B increases with time.
(vi) For t > t1 velocity of B is greater than velocity of A. Therefore after t = t1 separation between A &
B starts decreasing and it becomes zero at t = t2 where B overtakes A.

Now you can try Questions 14 to 38 in Exercise I and Ques. 7 to 11 in Ex.II

4. TWO DIMENSIONAL MOTION OR MOTION IN A PLANE


Motion in a plane can be described by vector sum of two independent 1D motions along two mutual
perpendicular directions (as motions along two mutual directions don’t affect each other).
Consider a particle moving in X-Y plane, then its equations of motions for X and Y axes are
vx = ux + axt, vy = uy + ayt
1 1
x = uxt + axt2, and ; y = uyt + ayt2, and
2 2
v 2v = u2x + 2a x x v 2y = u 2y + 2a y y
where symbols have their usual meanings. Thus resultant motion would be described by the equations
 
r = x i + y j and v = v i + v j
x y

394,50 - Rajeev Gandhi Nagar Kota, Ph. No. : 93141-87482, 0744-2209671


IVRS No : 0744-2439051, 52, 53, www. motioniitjee.com , hr@motioniitjee.com

Downloaded from www.iitjeephysics4u.com


KINEMATICS Page # 27

4.1 PROJECTILE MOTION


It is the best example to understand motion in a plane. If we project a particle obliquely from the
surface of earth, as shown in the figure below, then it can be considered as two perpendicular 1D
motions - one along the horizontal and other along the vertical.

Y u
usinθ
⇒ +
θ
u cos θ
O x
Assume that effect of air friction and wind resistance are negligible and value of ‘acceleration due to

gravity g is constant.
Take point of projection as origin and horizontal and vertical direction as +ve X and Y-axes, respectively.
For X-axis For Y - axis
ux = u cosθ, uy = u sinθ
ax = 0, ay = – g,
vx = u cosθ, and vy = u sinθ – gt, and
1
x = u cosθ × t y = u sinθ t – gt2
2
It is clear from above equations that horizontal component of velocity of the particle remains constant
while vertical component of velocity is first decreasing, gets zero at the highest point of trajectory and
then increases in the opposite direction. At the highest point, speed of the particle is minimum.
The time, which projectile takes to come back to same (initial) level is called the time of flight (T).
At initial and final points, y = 0,
1
So u sinθ t – gt2 = 0
2
2u sin θ 2u sin θ
⇒ t = 0 and t = So, T=
g g
Range (R) The horizontal distance covered by the projectile during its motion is said to be range of the
projectile
u 2 sin 2θ
R = u cosθ × T =
g
For a given projection speed, the range would be maximum for θ = 45°.
Maximum height attained by the projectile is
u 2 sin 2 θ
H=
2g
at maximum height the vertical component of velocity is 0.
u sin θ T
Time of ascent = Time of descent = =
g 2

Speed, kinetic energy, momentum of the particle initialy decreases in a projectile motion and attains a

minimum value (not equal to zero) and then again increases.


θ is the angle between v and horizontal which decreases to zero. (at top most point) and again

increases in the negative direction

394,50 - Rajeev Gandhi Nagar Kota, Ph. No. : 93141-87482, 0744-2209671


IVRS No : 0744-2439051, 52, 53, www. motioniitjee.com , hr@motioniitjee.com

Downloaded from www.iitjeephysics4u.com


Page # 28 KINEMATICS

Ex.33 A body is projected with a velocity of 30 ms–1 at an angle of 30° with the vertical. Find the
maximum height, time of flight and the horizontal range.
Sol. Here u = 30 ms–1,
Angle of projection, θ = 90 – 30 = 60°
Maximum height,
u 2 sin 2 θ 30 2 sin 2 60°
H= = = 34.44 m
2g 2 × 9.8
Time fo flight,
2u sin θ 2 × 30 sin 60°
T= = = 5.3 s
g 9.8
Horizontal range,
u 2 sin 2θ 30° sin 120° 30 2 sin 60°
R= = = = 79.53 m.
g 9.8 9.8
Ex.34 Find out the relation between uA, uB, uC (where uA, uB, uC are the initial velocities of particles A,
B, C, respectively)

B C
A

Sol. ∵ Hmax is same for all three particle A, B, C


u2y
⇒ Hmax =
2g
⇒ uy is same for all ∴ uyA = uyB = uyC

 2u y 
⇒ TA = TB = TC  g 
 

2u xu y
from figure RC > RB > RA ∵R =
g
⇒ uxC > uxB > uxA ⇒ uA < uB< uC

(C) Coordinate of a particle after a given time t :


Particle reach at a point P after time t then Y vy
x = ucosθ .t v
x α
vx
1 2 P(x,y)
y = usinθ.t – gt usinθ
2
u y
Position vector θ
O ucosθ X
  1 
r = (u cos θ.t ) î +  (u sin θ)t – gt 2  ĵ
 2 

(D) Velocity and direction of motion after a given time :


After time 't' vx = ucosθ and vy = usinθ – gt
2 2
Hence resultant velocity v = vx + vy = u2 cos 2 θ + (u sin θ – gt) 2

vy u sin θ – gt –1  u sin θ – gt 
tan α = = ⇒ α = tan  
vx u cos θ u cos θ 

394,50 - Rajeev Gandhi Nagar Kota, Ph. No. : 93141-87482, 0744-2209671


IVRS No : 0744-2439051, 52, 53, www. motioniitjee.com , hr@motioniitjee.com

Downloaded from www.iitjeephysics4u.com


KINEMATICS Page # 29

(E) Velocity and direction of motion at a given height :


At a height 'h', vx = ucosθ

And vy = u2 sin2 θ – 2gh


∴ Resultant velocity

v= v x2 + v y 2 = (u cos θ) 2 + u2 sin2 θ – 2gh

v= u2 – 2gh
Note that this is the velocity that a particle would have at height h if it is projected vertically from
ground with u.

Ex.35 A body is projected with a velocity of 20 ms–1 in a direction making an angle of 60° with the
horizontal. Calculate its (i) position after 0.5 s and (ii) velocity after 0.5 s.
Sol. Here u = 20 ms–1, θ = 60° , t = 0.5 s
(i) x = (u cosθ)t = (20 cos60°) × 0.5 = 5 m
1 2
y = (u sin θ) t – gt = (20 × sin 60°) × 0.5
2
1
– × 9.8 × (0.5)2 = 7.43 m
2
(ii) vx = u cos θ = 20 cos 60° = 10 ms–1
vy = u sin θ – gt = 20 sin 60° – 9.8 × 0.5
= 12.42 ms–1

vy 12.42
∴ v= v 2x + v 2y = (10) 2 + (12.42) 2 =15.95 ms
–1
tan β =
vx
= = 1.242
10
∴ β = tan–1 1.242 = 51.16°.

Equation of trajectory of a projectile.


Suppose the body reaches the point P(x, y) after time t.
Y
vy
v A Path of
x α v projectile
x

usinθ P(x,y)
Max.
u y height=h
θ
m
vx=u cosθ
O ucosθ X
B θ
R v
uy
∵ The horizontal distance covered by the body in time t,
x = Horizontal velocity × time = u cos θ. t
x
or t =
ucos θ
For vertical motion : u = u sinθ, a = –g, so the vertical distance covered in time t is given by

1 2 x 1 x2
s = ut + at or y = u sin θ. – g. 2
2 u cos θ 2 u cos 2 θ

1 x2
or y = x tanθ – g 2 ...(1)
2 u cos 2 θ

394,50 - Rajeev Gandhi Nagar Kota, Ph. No. : 93141-87482, 0744-2209671


IVRS No : 0744-2439051, 52, 53, www. motioniitjee.com , hr@motioniitjee.com

Downloaded from www.iitjeephysics4u.com


Page # 30 KINEMATICS

or y = px – qx2, where p and q are constants.


Thus y is a quadratic function of x. Hence the trajectory of a projectile is a parabola.

From equation (1)

 gx cos θ   gx 
y = x tan θ 1 – 2 2  ⇒ y = x tan θ 1 – 2u 2 cos θ sin θ 
 2u cos θ sin θ 

 x
y = x tan θ 1 –  ...(2)
 R

Equation (2) is another form of trajectory equation of projectile

Ex.36 A ball is thrown from ground level so as to just clear a wall 4 m high at a distance of 4 m and falls
at a distance of 14 m from the wall. Find the magnitude and direction of the velocity.
Sol. The ball passes through the point P(4, 4). So its range = 4 + 14 = 18m.
The trajectory of the ball is,
Now x = 4m, y = 4m and R = 18 m
y
 4  7
∴ 4 = 4 tan θ 1 –  = 4 tanθ . P(4,4)
 18  9 u

9 9 7
or tan θ = , sin θ = , cosθ = 4m
7 130 130
θ
18 × 9.8 × 130
or u2 = = 182 4m x
2×9×7 14m

or u = 182 = 13.5 ms
–1

Also θ = tan–1(9/7) = 52.1°

Ex.37 A particle is projected over a triangle from one end of a horizontal base and grazing the vertex
falls on the other end of the base. If α and β be the base angles and θ the angle of projection,
prove that tan θ = tan α + tan β.
Sol. If R is the range of the particle, then from the figure we have

y y y(R – x) + xy
tan α + tan β = + = Y
x R– x x(R – x)

y R
or tanα + tan β = × ...(1) P(x,y)
x (R – x)
Also, the trajectrory of the particle is
y
θ
 x α β
y = x tan θ 1–  O
x
 R x B A
R–x

y R
or tanθ = ×
x (R – x)
From equations (1) and (2), we get
tan θ = tan α + tan β .

394,50 - Rajeev Gandhi Nagar Kota, Ph. No. : 93141-87482, 0744-2209671


IVRS No : 0744-2439051, 52, 53, www. motioniitjee.com , hr@motioniitjee.com

Downloaded from www.iitjeephysics4u.com


KINEMATICS Page # 31

4.2 Projectile fired parallel to horizontal. As shown in shown u


O
figure suppose a body is projected horizontally with velocity u x
from a point O at a certain height h above the ground level. y
The body is under the influence of two simultaneous independent x P vx
motions: h β
(i) Uniform horizontal velocity u. v
(ii) Vertically downward accelerated motion with vy
constant acceleration g.
R
Under the combined effect of the above two motions,
Y A Ground
the body moves along the path OPA.
Trajectory of the projectile. After the time t, suppose the body reaches the point P(x, y).
The horizontal distance covered by the body in time t is
x
x = ut ∴ t=
u
The vertical distance travelled by the body in time t is given by
1 2
s = ut + at
2
1 2 1 2
or y=0×1+ gt = gt
2 2
[For vertical motion, u = 0]
2
1 x  g  2  x
or y= g  =  2 x ∵ t = u 
2 u  2u   
g
or y = kx2 [Here k = = a constant]
2u2
As y is a quadratic function of x, so the trajectory of the projectile is a parabola.
Time of flight. It is the total time for which the projectile remains in its flight (from 0 to A). Let T be its
time of flight.
For the vertical downward motion of the body,
we use
1 2
s = ut + at
2

1 2h
or h = 0 × T + gT2 or T=
2 g
Horizontal range. It is the horizontal distance covered by the projectile during its time of flight. It is
equal to OA = R. Thus R = Horizontal velocity × time of flight = u × T
2h
or R=u
g
Velocity of the projectile at any instant. At the instant t (when the body is at point P), let the
velocity of the projectile be v. The velocity v has two rectangular components:
Horizontal component of velocity, vx = u
Vertical component of velocity, vy = 0 + gt = gt
∴ The resultant velocity at point P is

v = v2x + v2y = u2 + g2 t2
If the velocity v makes an angle β with the horizontal, then
vy gt  gt 
tan β = = or β = tan–1  
vx u u

394,50 - Rajeev Gandhi Nagar Kota, Ph. No. : 93141-87482, 0744-2209671


IVRS No : 0744-2439051, 52, 53, www. motioniitjee.com , hr@motioniitjee.com

Downloaded from www.iitjeephysics4u.com


Page # 32 KINEMATICS

Ex.38 A body is thrown horizontally from the top of a tower and strikes the ground after three seconds
at an angle of 45° with the horizontal. Find the height of the tower and the speed with which
the body was projected. Take g = 9.8 ms–2.
Sol. As shown in figure, suppose the body is thrown horizontally from the top O of a tower of height y with
velocity u. The body hits the ground after 3s. Considering verticlly downward motion of the body,
1 2 1
y = uyt + gt = 0 × 3 + ×9.8 × (3)2 = 44.1 m [∴ Initial vertical velocity, uy = 0]
2 2
Final vertical velocity,
vy = uy + gt = 0 + 9.8 × 3 = 29.4 ms–1
Final horizontal velocity, vx = u
As the resultant velocity u makes an angle of 45° with the horizontal, so
vy 29.4
tan 45° = or 1 = or u = 29.4 ms–1.
vx x

Ex.39 A particle is projected horizontally with a speed u from the top of plane inclined at an angle θ
with the horizontal. How far from the point of projection will the particle strike the plane?
Sol. The horizontal distance covered in time t,
x
x = ut or t =
u u
The vertical distance covered in time t, θ
1 2 1 x2
y=0+ gt = g × 2 [using (1)] y D
2 2 u
y gx 2 θ
Also = tan θ or y = x tan θ ∴ = x tan θ
x 2u 2 x=ut
 gx 
or x  2 – tan θ  = 0
 2u 
2u 2 tan θ
As x = 0 is not possible, so x =
g
The distance of the point of strike from the point of projection is
D= x2 + y2 = x2 + (x tan θ)2
2u2
=x 1 + tan2 θ = x sec θ or D = tan θ sec θ
g

Ex.40 A ball rolls off the top of a stairway with a constant horizontal velocity u. If the steps are h
2hu 2
metre high and w meter wide, show that the ball will just hit the edge of nth step if n =
gw 2
Sol. Refer to figure. For n th step,
net vertical displacement = nh u
net horizontal displacement = nω 1st
Let t be the time taken by the ball to reach the nth step. Then 2nd
R = ut h
nω w
or nω = ut or t=
u
1 2
Also, y = uy t + gt nth
2 R
2
1 2 1  nω  2hu2
or nh = 0 + gt = g  or n =
2 2  u  gω2

394,50 - Rajeev Gandhi Nagar Kota, Ph. No. : 93141-87482, 0744-2209671


IVRS No : 0744-2439051, 52, 53, www. motioniitjee.com , hr@motioniitjee.com

Downloaded from www.iitjeephysics4u.com


KINEMATICS Page # 33

4.3 Projectile at an angle θ from height h


Consider the projectile as shown in the adjacent figure.
Take the point of projection as the origin the X and u
Y
Y-axes as shown in figure.
For X-axis, θ
u
x
= u cosθ
ax = 0
vx = u cosθ, and h
x = u cos θ × t
For Y-axis,
uy = u sin θ, x
ay = –g,
gt 2
vy = u sin θ – gt, and y = u sin θ t –
2
Ex.41 From the top of a tower 156.8 m high a projectile is projected with a velocity of 39.2 ms–1 in a
direction making an angle 30° with horizontal. Find the distance from the foot of tower where it
strikes the ground and time taken to do so.
Sol. The situation is shown –1
Here height of tower u=39.2 ms
OA = 156.8 m H
u = 39.2 ms–1
uy = usinθ
θ = 30°
time for which projectile remain is air = t = ?
Horizontal distance covered R = OD = ? θ=30°
A
Now ux = u cos θ and ux = u cosθ B

uy = u sin θ be the components of velocity u .
Motion of projectile from O to H to D
1 156.8 m
Using equation y = uyt + ay t2
2
Here : y = 156.8 m ; uy = – u sinθ
= 39.2 sin 30°
ay = 9.8 m/s2 ; t = ? O C D
156.8 = – 39.2 × 0.5 t + 4.9 t2
156.8 = – 19.6 t + 4.9 t2
or 4.9 t2 – 19.6 t – 156.8 = 0
or t2 – 4t – 32 = 0 ⇒ (t – 8) (t + 4) = 0
We get t = 8 s; t = – 4s
t = – 4 s is not possible, thus we take t = 8s.
Now horizontal distance covered in this time
R = ux × t = u cos θ × t = 39.2 × cos 30° × t
R = 271.57 m
4.4 Projectile Motion in Inclined Plane
Here, two cases arise. One is up the plane and the other is down the plane. Let us discuss both the
cases separately.
(i) Up the Plane : In this case direction x is chosen up the plane and direction y is chosen perpendicular
to the plane. Hence,
ux = u cos α , ax = – g sin β
uy = u sin α and ay = – g cos β

394,50 - Rajeev Gandhi Nagar Kota, Ph. No. : 93141-87482, 0744-2209671


IVRS No : 0744-2439051, 52, 53, www. motioniitjee.com , hr@motioniitjee.com

Downloaded from www.iitjeephysics4u.com


Page # 34 KINEMATICS

y x

B
u

gsinβ
β gcosβ
α
β g
β O
O
C
Now, let us derive the expressions for time of flight (T) and range (R) along the plane.

Time of flight
1
At point B displacement along y-direction is zero. So, substituting the proper values in sy = uyt + ay t2 ,
2
we get
1 2u sin α
α+ (– g cos β ) t2 ∴ t = 0 and
g cos β
0 = u t s i n
2
2u sin α
t = 0, corresponds to point O and t = corresponds to point B. Thus,
g cos β

2u sin α
T=
g cos β
Range

Range (R) or the distance OB is also equal to be displacement of projectile along x-direction in the
t = T. Therefore.
1 1
R = sa = uxT + axT2 ⇒ R = u cos α T – sin β T2
2 2

(ii) Down the inclined plane : y


along x - axis y-axis
(1) ux = ucos α (1) uy = usinα
(2) ax = g sin β (2) ay = g cos β
u sin α
u
velocity at P
vy = uy + ay T (0,0) α
vx = ux + axT uc
os
2u y 2u sin α α
Time of flight T = =
ay g cos β β
cos gs P
g β in
1 2 β
Range Sx = ux T + a x T
2 β
1 g
2
= u cos α T + g sin β. T
2 x

394,50 - Rajeev Gandhi Nagar Kota, Ph. No. : 93141-87482, 0744-2209671


IVRS No : 0744-2439051, 52, 53, www. motioniitjee.com , hr@motioniitjee.com

Downloaded from www.iitjeephysics4u.com


KINEMATICS Page # 35

Ex.42 A particle is projected at an angle α with horizontal from the foot of a plane whose inclination to
horizontal is β . Show that it will strike the plane at right angles if cotβ = 2 tan (α – β)
Sol. Let u be the velocity of projection so that u cos (α – β ) and u sin (α – β ) are the initial velocities
respectively parallel and perpendicular to the inclined plane. The acceleration in these two directions
are (–g sin β ) and (–g cos β ).
The initial component of velocity perpendicular to PQ is u sin (α – β ) and the acceleration in this
direction is (–g cosβ ). If T is the time the particle takes to go from P to Q then in time T the space
described in a direction perpendicular to PQ is zero.
1 u
0 = u sin (α – β ).T – g cos β .T2 Q
2
2u sin(α – β)
T=
g cos β
α
If the direction of motion at the instant when the particle β
hits the plane be perpendicular to the plane, then the
velocity at that instant parallel to the plane must be zero. P N
∴ u cos (α – β ) – g sin β T = 0
u cos(α – β) 2u sin(α – β)
=T=
g sin β g cos β
∴ cosβ = 2 tan (α – β )

Ex.43 Two inclined planes OA and OB having inclinations 30° and 60° with x
y
the horizontal respectively intersect each other at O, as shwon in
u v B
figure. a particle is projected from point P with velocity u = 10 3 m / s
along a direction perpendicular to plane OA. If the particle strikes A Q

plane OB perpendicular of flight, then calculate. P


h
30° 60°
(a) time of flight
O
(b) velocity with which the particle strikes the plane OB,
(c) height h of point P from point O
(d) distance PQ. (Take g = 10 m/s2)
Sol. Let us choose the x and y directions along OB and OA respectively. Then,
ux = u = 10 3 m/s, uy = 0
ax = – g sin 60° = – 5 3 m/s2
and ay = – g cos 60° = – 5 m/s2
(a) At point Q, x-component of velocity is zero. Hence, substituting in
vx= ux + axt
10 3
0 = 10 3 – 5 3t ⇒ t= = 2s Ans.
5 3
(b) At point Q, v = vy = uy + ayt
∴ v = 0 – (5) (2) = –10 m/s Ans.
Here, negative sign implies that velocity of particle at Q is along negative y direction.
(c) Distance PO = |displacement of particle along y-direction| = |sy|
1 1
Here, sy = uyt + ay t2 = 0 – (5)(2)2 = – 10 m
2 2
∴ PO = 10 m

394,50 - Rajeev Gandhi Nagar Kota, Ph. No. : 93141-87482, 0744-2209671


IVRS No : 0744-2439051, 52, 53, www. motioniitjee.com , hr@motioniitjee.com

Downloaded from www.iitjeephysics4u.com


Page # 36 KINEMATICS

1
Therefore, h = PO sin 30° = (10)   or h = 5m Ans.
2

(d) Distance OQ = displaement of particle along


x-direction = sx
1 1
Here, sx = uxt + ax t2 = (10 3)(2) – (5 3)(2)2 = 10 3 m
2 2

or OQ = 10 3 m

PQ = (PO)2 + (OQ)2 = (10)2 + (10 3)2 = 100 + 300 = 400


PQ = 20 m Ans.

Now you can try Questions 45 to 68 in Exercise I and Ques. 12 to 20 in Ex.II

5. RELATIVE MOTION

The word 'relative' is a very general term, which can be applied to physical, nonphysical, scalar or
vector quantities. For example, my height is five feet and six inches while my wife's height is five feet
and four inches. If I ask you how high I am relative to my wife, your answer will be two inches. What
you did? You simply subtracted my wife's height from my height. The same concept is applied everywhere,
whether it is a relative velocity, relative acceleration or anything else. So, from the above discussion

we may now conclude that relative velocity of A with respect of B (written as v AB ) is

→ → →
v AB = v A – v B
Similarly, relative acceleration of A with respect of B is
→ → →
a AB = a A – a B
If it is a one dimensional motion we can treat the vectors as scalars just by assigning the positive sign
to one direction and negative to the other. So, in case of a one dimensional motion the above
equations can be written as
vAB = vA – vB
and aAB = aA – aB
Further, we can see that
→ → → →
v AB = – v BA or a BA = – a AB

Ex.44 Seeta is moving due east with a velocity of 1 m/s and Geeta is moving the due west with a
velocity of 2 m/s. What is the velocity of Seeta with respect to Geeta?
Sol. It is a one dimensional motion. So, let us choose the east direction as positive and the west as
negative. Now, given that
vs = velocity of Seeta = 1 m/s
and vG = velocity of Geeta = – 2m/s
Thus, vSG = velocity of Seeta with respect to Geeta
= vS – vG = 1 – (–2) = 3 m/s
Hence, velocity of Seeta with respect to Geeta is 3 m/s due east.

394,50 - Rajeev Gandhi Nagar Kota, Ph. No. : 93141-87482, 0744-2209671


IVRS No : 0744-2439051, 52, 53, www. motioniitjee.com , hr@motioniitjee.com

Downloaded from www.iitjeephysics4u.com


KINEMATICS Page # 37

IMPORTANT NOTE :
PROCEDURE TO SOLVE THE VECTOR EQUATION.
  
A =B+C ...(1)
(a) Their are 6 variables in this equation which are following :

(1) Magnitude of A and its direction

(2) Magnitude of B and its direction

(3) Magnitude of C and its direction.
(b) We can solve this equation if we know the value of 4 varibales [Note : two of them must be directions]
(c) If we know the two direction of any two vectors then we will put them on the same side and other on
the different side.
For example
  
If we know the directions of A and B and C' s direction is unknown then we make equation as follows : -
  
C = A –B
(d) Then we make vector diagram according to the equation and resolve the vectors to know the
unknown values.

Ex.45 Car A has an acceleration of 2 m/s2 due east and car B, 4 m/s2 due north. What is the acceleration
of car B with respect to car A?
Sol. It is a two dimensional motion. Therefore, N

a BA = acceleration of car B with respect to car A
→ → W E
= aB = – a A

Here, a B = acceleration of car
S
B = 4 m/s2 (due north)

and a A = acceleration of car A = 2 m/s2 (due east)
→ →
→ a BA a B = 4m / s 2
| a BA |= (4)2 + (2)2 = 2 5m / s2

4
and α = tan–1   = tan–1(2)
2 α
→ →
Thus, a BA is 2 5 m/s2 at an angle of α = tan–1(2) – a A = 2m / s 2
from west towards north.

Ex.46 Three particle A, B and C situated at the vertices of an equilateral triangle starts moving simul-
taneously at a constant speed "v" in the direction of adjacent particle, which falls ahead in the
anti-clockwise direction. If "a" be the side of the triangle, then find the time when they meet.
A
Sol. Here, particle "A" follows "B", "B" follows "C" and "C" follows
"A". The direction of motion of each particle keeps chang-
ing as motion of each particle is always directed towards
other particle. The situation after a time "t" is shown in the
figure with a possible outline of path followed by the par- O
ticles before they
meet. B C

394,50 - Rajeev Gandhi Nagar Kota, Ph. No. : 93141-87482, 0744-2209671


IVRS No : 0744-2439051, 52, 53, www. motioniitjee.com , hr@motioniitjee.com

Downloaded from www.iitjeephysics4u.com


Page # 38 KINEMATICS

This problem appears to be complex as the path of motion


is difficult to be defined. But, it has a simple solution in A
component analysis. Let us consider the pair "A" and "B".
The initial component of velocities in the direction of line v
joining the initial position of the two particles is "v" and
"vcosθ" as shown in the figure here :
The component velocities are directed towards eachother. v cos θ O v
Now, considering the linear (one dimensional) motion in the 60°
direction of AB, the relative velocity of "A" with respect to B C
v
"B" is :
vAB = vA – vB
vAB = v – (– v cos θ) = v + vcosθ
In equilateral triangle, θ = 60°
v 3v
vAB = v + vcos60° = v + =
2 2
The time taken to cover the displacement "a" i.e. the side of the triangle
2a
t=
3v
QUESTIONS BASED ON RELATIVE MOTION ARE USUALLY OF FOLLOWING FOUR TYPES :
(a) Minimum distance between two bodies in motion
(b) River-boat problems
(c) Aircraft-wind problems
(d) Rain problems
(a) Minimum distance between two bodies in motion

When two bodies are in motion, the questions like, the minimum distance between them or the time
when one body overtakes the other can be solved easily by the principle of relative motion. In these
type of problems one body is assumed to be at rest and the relative motion of the other body is
considered. By assuming so two body problem is converted into one body problem and the solution
becomes easy. Following example will illustrate the statement.

Ex.47 Car A and car B start moving simultaneously in the same direction along the line joining them.
Car A with a constant acceleration a = 4 m/s2, while car B moves with a constant velocity v = 1
m/s. At time t = 0, car A is 10 m behind car B. Find the time when car A overtakes car B.
Sol. Given : uA = 0, uB = 1 m/s, aA = 4m/s2 and aB = 0
Assuming car B to be at rest, we have
uAB = uA – uB = 0 – 1 = – 1 m/s
aAB = aA – aB = 4 – 0 = 4 m/s2
Now, the problem can be assumed in simplified form as follow :
2 2
a=4m/s v=1m/s
A 10m B

+ve
Substituting the proper values in equation
2
uAB= –1m/s, aAB= 4m/s
A 10m B

At rest

394,50 - Rajeev Gandhi Nagar Kota, Ph. No. : 93141-87482, 0744-2209671


IVRS No : 0744-2439051, 52, 53, www. motioniitjee.com , hr@motioniitjee.com

Downloaded from www.iitjeephysics4u.com


KINEMATICS Page # 39

1 2
s = ut + at
2
1
we get 10 = – t + (4)(t2 ) or 2t2 – t – 10 = 0
2
1 ± 1 + 80 1 ± 81 1±9
or t= = = or t = 2.5 s and – 2 s
4 4 4
Ignoring the negative value, the desired
time is 2.5s. Ans.
Note : The above problem can also be solved without using the concept of relative motion as under.
At the time when A overtakes B,
sA = sB + 10
1
∴ × 4 × t 2 = 1 × t + 10
2
or 2t2 – t – 10 = 0
Which on solving gives t = 2.5 s and – 2 s, the same as we found above.
As per my opinion, this approach (by taking absolute values) is more suitable in case of two body
problem in one dimensional motion. Let us see one more example in support of it.

Ex.48 An open lift is moving upwards with velocity 10m/s. It has an upward acceleration of 2m/s2. A
ball is projected upwards with velocity 20 m/s relative to ground. Find :
(a) time when ball again meets the lift.
(b) displacement of lift and ball at that instant.
(c) distance travelled by the ball upto that instant. Take g = 10 m/s2
Sol. (a) At the time when ball again meets the lift,
sL = sB
1 1 2m/s2 10m/s 20m/s
∴ 10t + × 2 × t2 = 20 t – × 10t2
2 2 +ve
Solving this equation, we get Ball
2
5 10m/s
t=0 and t= s
3 L Lift B Ball

5
∴ Ball will again meet the lift after s.
3
(b) At this instant
2
5 1 5 175
sL = sB = 10 × + ×2×  = m = 19.4 m
3 2 3
  9

(c) For the ball u ↑ ↓a . Therefore, we will first find t0, the time when its velocity becomes zero.

u 20
t0 = = = 2s
a 10

 5 
As t  = s  < t0 , distance and displacement are equal
 3 
or d = 19.4 m Ans.
Concept of relative motion is more useful in two body problem in two (or three) dimensional motion.
This can be understood by the following example.

394,50 - Rajeev Gandhi Nagar Kota, Ph. No. : 93141-87482, 0744-2209671


IVRS No : 0744-2439051, 52, 53, www. motioniitjee.com , hr@motioniitjee.com

Downloaded from www.iitjeephysics4u.com


Page # 40 KINEMATICS

Ex.49 Two ships A and B are 10 km apart on a line running south to north. Ship A farther north is
streaming west at 20 km/h and ship B is streaming north at 20km/h. What is their distance
of closest approach and how long do they take to reach it ?
Sol. Ships A and B are moving with same speed 20 km/h in
the directions shown in figure. It is a two dimensional, N
two body problem with zero acceleration. Let us find vA A E

vBA
   vB
vBA = vB − v A
 B
Here, | vBA |= (20)2 + (20)2 AB=10km

= 20 2 km / h


i.e., vBA is 20 2 km / h at an angle of 45º from east
towards north. Thus, the given problem can be simplified as :
45º


A is at rest and B is moving with vBA in the direction shown in figure. A
Therefore, the minimum distance between the two is C
vBA
smin = AC = AB sin 45º 45º
B
 1 
= 10   km = 5 2 km Ans.
 2
and the desired time is
BC 5 2
t=  = (BC = AC = 5 2 km )
| vBA | 20 2
1
= h = 15 min Ans.
4

(B) River - Boat Problems

In river-boat problems we come across the following three terms :


B B
→ v br cosθ
v br W y
θ θ
A → x
vr vbr sinθ A
vr

v r = absolute velocity of river

v br = velocity of boatman with respect to river or velocity of boatman is still water
and
→ = absolute velocity of boatman.
vb
→ →
• Here, it is important to note that v br is the velocity of boatman with which he steers and v b is the
actual velocity of boatman relative to ground.
→ → →
Further, v b = v br + v r
Now, let us derive some standard results and their special cases.

394,50 - Rajeev Gandhi Nagar Kota, Ph. No. : 93141-87482, 0744-2209671


IVRS No : 0744-2439051, 52, 53, www. motioniitjee.com , hr@motioniitjee.com

Downloaded from www.iitjeephysics4u.com


KINEMATICS Page # 41


A boatman starts from point A on one bank of a river with velocity v br in the direction shown in fig.

River is flowing along positive x-direction with velocity v r . Width of the river is w, then
→ → →
v b = v br + v r
Therefore, vbx = vrx + vbrx = vr – vbr sinθ
and vby = vry + vbry
= 0 + vbr cosθ = vbr cosθ
Now, time taken by the boatman to cross the river is :
w w
t= =
v by v br cos θ
w
or t = v cos θ ...(i)
br
Further, displacement along x-axis when he reaches on the other bank (also called drift) is :
w
x = vbx t = (vr – vbr sin θ) v cos θ
br
w
or x = (vr – vbr sinθ) v cos θ ...(ii)
br
Three special are :

(i) Condition when the boatman crosses the river in shortest interval of time
B
From Eq.(i) we can see that time (t) will be minimum when θ = 0°,
i.e., the boatman should steer his boat perpendicular to the river →
current. vbr
w
Also, tmin = v as cos θ = 1 A →
br
vr
(ii) Condition when the boatman wants to reach point B, i.e., at a point just opposite from where
he started
In this case, the drift (x) should be zero.
∴ x=0 B
w →
or (vr – vbr sinθ) v cos θ = 0 v br
br
θ
or vr = vbr sin θ
A →
–1  v 
vr vr
or sinθ = v or θ = sin  r 
br  v br 

–1  v 
Hence, to reach point B the boatman should row at an angle θ = sin  r  upstream from AB.
 v br 
Further, since sinθ not greater than 1.
So, if vr ≥ vbr, the boatman can never reach at point B. Because if vr = vbr, sin θ = 1 or θ = 90° and it is
just impossible to reach at B if θ = 90°. Moreover it can be seen that vb = 0 if vr = vbr and θ = 90°.
Similarly, if vr > vbr, sinθ > 1, i.e., no such angle exists. Practically it can be realized in this manner that
it is not possible to reach at B if river velocity (vr) is too high.
(iii) Shortest path
Path length travelled by the boatman when he reaches the opposite shore is
s= w 2 + x2
Here, w = width of river is constant. So for s to be minimum modulus of x (drift) should be minimum.
Now two cases are possible.

394,50 - Rajeev Gandhi Nagar Kota, Ph. No. : 93141-87482, 0744-2209671


IVRS No : 0744-2439051, 52, 53, www. motioniitjee.com , hr@motioniitjee.com

Downloaded from www.iitjeephysics4u.com


Page # 42 KINEMATICS

When vr < vbr : In this case x = 0,

–1  v r  –1  v r 
when θ = sin  v  or smin = w at θ = sin  
 br   v br 
dx
When vr > vbr : In this case x is minimum, where =0

d  w 
or  (vr – vbr sin θ) = 0
dθ  vbr cos θ 
or –vbr cos2θ – (vr – vbr sinθ) (– sinθ) = 0
or – vbr + vr sinθ = 0
 vbr 
or θ = sin–1  
 vr 
Now, at this angle we can find xmin and then smin which comes out to be
 vr  –1  vbr 
smin = w   at θ = sin  
 vbr   vr 

Ex.50 A man can row a boat with 4 km/h in still water. If he is crossing a river where the current is 2
km/h.
(a) In what direction will his boat be headed, if he wants to reach a point on the other bank, directly
opposite to starting point?
(b) If width of the river is 4 km, how long will the man take to cross the river, with the condition in
part (a)?
(c) In what direction should he head the boat if he wants to cross the river in shortest time and
what is this minimum time?
(d) How long will it take him to row 2 km up the stream and then back to his starting point ?
Sol. (a) Given, that vbr = 4 km/h and vr = 2 km/h

 vr  2 1
∴ θ = sin–1  v  = sin–1   = sin–1   = 30°
 br  4
  2
Hence, to reach the point directly opposite to starting point he should head the boat at an angle of
30° with AB or 90° + 30° = 120° with the river flow.
(b) Time taken by the boatman to cross the river
w = width of river = 4 km
vbr = 4 km/h and θ = 30°
4 2
∴ t= = h Ans.
4 cos 30° 3
(c) For shortest time θ = 0°
w 4
and tmin = v cos 0° = = 1h
br 4
Hence, he should head his boat perpendicular to the river current for crossing the river in shortest time
and this shortest time is 1 h.

vbr–vr vbr+vr
D C D C

(d) t = tCD + tDC


CD DC 2 2 1 4
or t= + = + = 1+ = h Ans.
v db – v r v br + v r 4–2 4+2 3 3

394,50 - Rajeev Gandhi Nagar Kota, Ph. No. : 93141-87482, 0744-2209671


IVRS No : 0744-2439051, 52, 53, www. motioniitjee.com , hr@motioniitjee.com

Downloaded from www.iitjeephysics4u.com


KINEMATICS Page # 43

Ex.51 A man can swim at a speed of 3 km/h in still water. He wants to cross a 500 m wide river
flowing at 2 kh/h. He keeps himself always at an angle of 120° with the river flow while swim-
ming.
(a) Find the time he takes to cross the river.
(b) At what point on the opposite bank will he arrive ?
Sol. The situation is shown in figure

Here vr,g = velocity of the river with respect to the ground
Y
 B C
vm,r = velocity of the man with respect to the river

vm,g = velocity of the man with respect to the ground.

,g
vm
(a) We have

°
  

30
vm,g = vm,r + vr,g ...(i) vm,r = 3km/h
θ
Hence, the velocity with respect to the ground is along AC.
Taking y-components in equation (i),
A vr,g = 2km/h
 3 3
vm,g sinθ = 3 km/h cos 30° + 2 km/h cos 90° = km/h
2
Time taken to cross the river
displacement along the Y - axis 1/ 2km 1
= = = h
velocity along the Y - axis 3 3 / 2 km / h 3 3
(b) Taking x-components in equation (i),
 1
vm,g cos θ = –3km/h sin 30° + 2 km/h = km / h
2
Displacement along the X-axis as the man crosses the river
= (velocity along the X-axis) (time)

 1km   1  1
=   ×  h = km
2h  3 3  6 3

Ex.52 A boat moves relative to water with a velocity v and river is flowing with 2v. At what angle the
boat shall move with the stream to have minimum drift?
(A) 30° (B) 60° (C) 90° (D) 120°
Sol. (D) Let boat move at angle θ to the normal as shown in
1
figure then time to cross the river =
v cos θ
ucos θ
1 ub = u
drift x = (2v – v sin θ) for x to be minimum I = width of river
v cos θ
dx
= 0 = 1 (2 sec θ tan θ – sec2θ) or sin θ = 1/2

u sinθ ur=2v
or θ = 30° and φ = 90 + 30 = 120°

(C) Aircraft Wind Problems


→ →
This is similar to river boat problem. The only difference is that v br is replaced by v aw (velocity of
→ →
aircraft with respect to wind or velocity of aircraft in still air), v r is replaced by v w (velocity of wind)

→ → → → →
and v b is replaced by v a (absolute velocity of aircraft). Further,, v a = v aw + v . The following
w
example will illustrate the theory.

394,50 - Rajeev Gandhi Nagar Kota, Ph. No. : 93141-87482, 0744-2209671


IVRS No : 0744-2439051, 52, 53, www. motioniitjee.com , hr@motioniitjee.com

Downloaded from www.iitjeephysics4u.com


Page # 44 KINEMATICS

NOTE : SHORT - TRICK 


A   
C = A +B
 
If their are two vectors A and B and their resultent
 
make an anlge α with A and β with B . α
then A sin α = β sin β A sinα β 
 B
Means component of A perpendicular to resultant is equal in B sinβ

magnitude to the compopent of B also perpendicular to resultant.

  
Ex.53 If two vectors A and B make angle 30° and 60° B

with their resultent and B has magnitude equal to
 60°
10, then find magnitude of A . 30° 
Bsin60° A
So B sin 60° = A sin 30°
⇒ 10 sin 60° = A sin 30° A sin 30°
⇒ A = 10 3

Ex.54 An aircraft flies at 400 km/h in still air. A wind of 200 2 km/h is blowing from the south. The
pilot wishes to travel from A to a point B north east of A. Find the direction he must steer
and time of his journey if AB = 1000 km.
Sol. Given that vw = 200 2 km/h
→ →
vaw = 400 km/h and v a should be along AB or in north-east direction. Thus, the direction of v aw
→ →
should be such as the resultant of v w and v aw is along AB or in north - east direction.
→ N
Let v aw makes an angle α with AB as shown in figure.
B
Applying sine law in triangle ABC, we get

AC
=
BC v a 45° → v w = 200 2km / h
sin 45° sin α
 BC   200 2  1 1 45°
α → C
sin α =  
 sin 45° =   = A v aw = 400 km / h
or
AC  
 400  2 2 E
∴ α = 30°
Therefore, the pilot should steer in a direction at an angle of (45° + α) or 75° from north towards
east.

| v a| 400 → sin 105° km
Further, = or | v a | = sin 45° × (400) h
sin(180°–45°–30° ) sin 45°

 cos 15°  km  0.9659  km


=   (400) =   (400)
sin 45°  h 0.707  h
= 546.47 km/h
∴ The time of journey from A to B is
AB 1000
t= → = h ⇒ t = 1.83 h
546.47
| v a|
(D) Rain Problems
→ → →
In these type of problems we again come across three terms v r , vm and vrm , Here,

v r = velocity of rain

394,50 - Rajeev Gandhi Nagar Kota, Ph. No. : 93141-87482, 0744-2209671


IVRS No : 0744-2439051, 52, 53, www. motioniitjee.com , hr@motioniitjee.com

Downloaded from www.iitjeephysics4u.com


KINEMATICS Page # 45


vm = velocity of man (it may be velocity of cyclist or velocity of motorist also)


and vrm = velocity of rain with respect to man.


Here, v is the velocity of rain which appears to the man. Now, let us take one example of this.
rm

Ex.55 A man standing on a road has to hold his umbrella at 30° with the vertical to keep the rain
away. He throws the umbrella and starts running at 10 km/h. He finds that raindrops are hitting
his head vertically. Find the speed of raindrops with respect to (a) the road, (b) the moving man.
Sol. When the man is at rest with respect to the ground, the rain comes to him at an angle 30° with the
vertical. This is the direction of the velocity of raindrops with respect to the ground. The situation
when the man runs is shown in the figure

30° vm,g

30°

v r,m vr,g
(b)
(a)


Here vr,g = velocity of the rain with respect to the ground
 
vm,g = velocity of the man with respect to the ground and vr,m = velocity of the rain with respect to
the man.
  
We have, vr,g = vr,m + vm,g ...(i)
Taking horizontal components, equation (i) gives

10 km / h
vr,g sin30° = um,g = 10 km/h or, v,g = = 20km / h
sin 30°
Taking vertical components, equation (i) gives

3
vr,g cos30° = vr,m or, vr,m = (20 km/h) = 10 √ 3 km/h.
2
Ex.56 To a man walking at the rate of 3 km/h the rain appears to fall vertically. When the increases
his speed to 6 km/h it appears to meet him at an angle of 45° with vertical. Find the speed of rain.
Sol. Let i and j be the unit vectors in horizontal and vertical directions respectively..
Let velocity of rain Vertical ( j )

v r = aiˆ + bj
ˆ ...(i)
Then speed of rain will be

| v r |= a2 + b2
Horizontal ( i )

394,50 - Rajeev Gandhi Nagar Kota, Ph. No. : 93141-87482, 0744-2209671


IVRS No : 0744-2439051, 52, 53, www. motioniitjee.com , hr@motioniitjee.com

Downloaded from www.iitjeephysics4u.com


Page # 46 KINEMATICS


In the first case v m = velocity of man = 3 i
→ → →
∴ v rm = v r – v m = (a – 3)iˆ + bj
ˆ
It seems to be in vertical direction. Hence,
a – 3 = 0 or a = 3

In the second case v m = 6 i

→ 
∴ ˆ = – 3 i + b j
v rm = (a – 6)iˆ + bj
This seems to be at 45° with vertical.
Hence, |b| = 3
Therefore, from Eq. (ii) speed of rain is

| v r |= (3)2 + (3)2 = 3 2 km / h Ans.

Relative Motion between Two Projectiles


Let us now discuss the relative motion between two projectiles or the path observed by one projectile
of the other. Suppose that two particles are projected from the ground with speeds u1 and u2 at angles
α1 and α2 as shown in Fig.A and B. Acceleration of both the particles is g downwards. So, relative
acceleration between them is zero because
a12 = a1 – a2 = g – g = 0

Y Y
u1 u2

α1 α2
X X
(A) (B)

i.e., the relative motion between the two particles is uniform. Now
u1x = u1 cos α1, u2x = u2 cos α2
u1y = u1 sin α1 and u2y = u2 sin α2
Therefore, u12x = u1x – u2x = u1 cos α1– u2cos α2 y
and u12y = u1y – u2y = u1 sin α1– u2 sin α2
u12x and u12y are the x and y components of relative
u12y
velocity of 1 with respect to 2.
Hence, relative motion of 1 with respect to 2 is a straight u12
a12=0
θ
u  x
line at an angle θ = tan −1 12 y  with positive x-axis. u12x
 u12 x 

Now, if u12x = 0 or u1 cos α1 = u2 cos α2, the relative motion is along y-axis or in vertical direction
(as θ = 90º). Similarly, if u12y = 0 or u1 sin α1 = u2 sin α2, the relative motion is along x-axis or in
horizontal direction (as θ = 0º).
Note : Relative acceleration between two projectiles is zero. Relative motion between them is uniform.
Therefore, condition of collision of two particles in air is that relative velocity of one with respect to

the other should be along line joining them, i.e., if two projecticles A and B collide in mid air, then VAB

should be along AB or VBA along BA.

394,50 - Rajeev Gandhi Nagar Kota, Ph. No. : 93141-87482, 0744-2209671


IVRS No : 0744-2439051, 52, 53, www. motioniitjee.com , hr@motioniitjee.com

Downloaded from www.iitjeephysics4u.com


KINEMATICS Page # 47

Condition for collision of two projectiles : Consider the situation shown in the figure. For projectiles to
collide, direction of velocity of A with respect to B has to be along line AB.
Here, vABx = u1 cos α1 + u2 cos α2
u2
vABy = u1 sin α1 – u2 sin α2 B
Y
Let, direction of velocity vector of A(wrt B) is making an u1
h2
angle β with +ve X-axis, which is given by X
A
v ABy u1 sin α1 − u2 sin α2 h1
tan β = =
v ABx u1 cos α1 + u2 cos α2
x
For collision to take place,
h2 − h1
tan β = tan θ =
x
Ex.57 A particle A is projected with an initial velocity of 60 m/s. at an angle 30º to the horizontal. At
the same time a second particle B is projected in opposite direction with initial speed of 50 m/s
from a point at a distance of 100 m from A. If the particles collide in air, find (a) the angle of
projection α of particle B, (b) time when the collision takes place and (c) the distance of P from A,
where collision occurs. (g = 10 m/s2)
60m/s 50m/s

30º
A B
100m
Sol. (a) Taking x and y directions as
shown in figure. Y
 
Here, a = −gˆj , a = −gˆj
A B

uAx = 60 cos 30º = 30 3 m / s


uAy = 60 sin 30º = 30 m/s X
uBx = – 50 cos α u AB

and uBy = 50 sin α


 
Relative acceleration between the two is zero as a A = aB . Hence, the relative motion between the two

is uniform. It can be assumed that B is at rest and A is moving with u AB . Hence, the two particles will

collide, if u AB is along AB. This is possible only when
uAy = uBy
i.e., component of relative velocity along y-axis should be zero.
or 30 = 50 sin α
∴ = sin–1 (3/5)
a Ans.

  4
(b) Now, | u AB |= u Ax – uBx = (30 3 + 50 cosα)m/s =  30 3 + 50 ×  m / s = (30 3 + 40) m/s
 5

Therefore, time of collision is

AB 100
t=  = or t = 1.09 s Ans.
| u AB | 30 3 + 40
(c) Distance of point P from A where collision takes place is
2 2
 1   1 
s= (u Ax t) 2 +  u Ay t – gt 2  = . ) 2 +  30 × 109
( 30 3 × 109 . – × 10 × 109
. × 109
.  or s = 62.64 m Ans.
 2   2 

394,50 - Rajeev Gandhi Nagar Kota, Ph. No. : 93141-87482, 0744-2209671


IVRS No : 0744-2439051, 52, 53, www. motioniitjee.com , hr@motioniitjee.com

Downloaded from www.iitjeephysics4u.com


Page # 48 KINEMATICS

5m/s
Ex.58 Two projectile are projected simultaneously from a point on the A
ground "O" and an elevated position "A" respectively as shonw in the
figure. If collision occurs at the point of return of two projectiles on
H
the horizontal surface, then find the height of "A" above the ground 10m/s
and the angle at which the projectile "O" at the ground
should be projected.
θ x
O C

Sol. There is no initial separation between two projectile is x-direction. For collision to occur, the relative
motion in x-direction should be zero. In other words, the component velocities in x-direction should be
equal to that two projetiles cover equal horizontal distance at any given time. Hence,
uOx = uAx

uA 5 1
⇒ u0cosθ = uA ⇒ cosθ = = = = cos60° ⇒ θ = 60°
uO 10 2
We should ensure that collision does occur at the point of return. It means that by the time projectiles
travel horizontal distances required, they should also cover vertical distances so that both projectile are
at "C" at the same time. In the nutshell, their times of flight should be equal.
For projectile from "O".
2uO sin θ
T=
g
For projectile from "A",
 2H 
T=  
 g 
For projectile from "A",
2uo sin θ  2H 
T= =  
g  g 
Squaring both sides and putting values,
4u2O sin2 θ 4 × 102 sin2 60°
⇒ H= ⇒ H=
2g 2 × 10
2
 3
H = 20  = 15m
 2 
 
We have deliberately worked out this problem taking advantage of the fact that projectiles are colliding
at the end of their flights and hence their times of flight should be equal. We can, however, proceed to
analyze in typical manner, using concept of relative velocity. The initial separation between two projectiles
in the vertical direction is "H". This separation is covered with the component of relative in vertical
direction.
3
⇒ vOAy = uOy – uAy = u0 sin60° – 0 = 10 × = 5 3m/s
2
Now, time of flight of projectile from ground is :
2uO sin θ 2x10x sin 60°
T= = = 3
g 10
Hence, the vertical displacement of projectile from "A" before collision is :

⇒ H = vOAy X T = 5 3 x 3 = 15 m/s

394,50 - Rajeev Gandhi Nagar Kota, Ph. No. : 93141-87482, 0744-2209671


IVRS No : 0744-2439051, 52, 53, www. motioniitjee.com , hr@motioniitjee.com

Downloaded from www.iitjeephysics4u.com


KINEMATICS Page # 49

Ex.59 Two projectiles are projected simultaneously from two towers as shwon in figure. If the projectiles
collide in the air, then find the distance "s" between the towers.

10 m/s B

10 2 m / s

30m
A 45°

10m

Sol. We see here that projectiles are approaching both horizontally and vertically. Their movement in two
component directions should be synchronized so that they are at the same position at a particular given
time. For collision, the necessary requirement is that relative velocity and displacement should be in the
same direction.
It is given that collision does occur. It means that two projectiles should cover the displacement with
relative velocity in each of the component directions. Y 10 m/s B
In x-direction,

1 10 2 m / s
vABx = uAx – uBx = 10 2 cos 45° – (–10) = 10 2 + 10 = 20 m/s
2 30m
A 45°
If "t" is time after which collision occurs, then
⇒ s = vAy – uBy 10m

1
⇒ vABy = ucos45° – 0 = 10 2 × = 10m / s O S x
2
The initial vertical distance between points of projection is 30 – 10 = 20 m. This vertical distance is
covered with component of relative velocity in vertical direction. Hence, time taken to collide, "t", is :

20
⇒t= =2
10
Putting this value in the earlier equation for "s", we have :
⇒ s = 20t = 20x2 = 40 m

Now you can try all the questions related to relative motion.

394,50 - Rajeev Gandhi Nagar Kota, Ph. No. : 93141-87482, 0744-2209671


IVRS No : 0744-2439051, 52, 53, www. motioniitjee.com , hr@motioniitjee.com

Downloaded from www.iitjeephysics4u.com


Page # 50 KINEMATICS

QUESTIONS FOR SHORT ANSWER



1. A vector a is turned without a change in its length 5. Can you have zero displacement and a non zero
 average velocity? Can you have a zero displacement
through a small angle dθ. What are | a| and ∆a?
and a non zero velocity? Illustrate your answer on a
Sol.
x-t graph.
Sol.

2. Does the speedometer of a car measure speed or


velocity ? Explain
6. At which point on its path a projectile has the
Sol.
smallest speed?
Sol.

7. A person standing on the edge of a cliff at some


3. When a particle moves with constant velocity, its
height above the ground below throws one ball straight
average velocity and its instantaneous velocity & speed
up with initial speed u and then throws another ball
are equal. Comment on this statement.
straight down with the same initial speed. Which ball,
Sol.
if either, has the larger speed when it hits the ground?
Neglect air resistance.
Sol.

4. In a given time interval, is the total displacement


8. An airplane on floor relief mission has to drop a
of a particle equal to the product of the average
sack of rice exactly in the center of a circle on the
velocity and the time interval, even when the velocity
ground while flying at a predetermined height and
is not constant? Explain.
speed. What is so difficult about that? Why doesn’t it
Sol.
just drop the sack when it is directly above the circle.
Sol.

394,50 - Rajeev Gandhi Nagar Kota, Ph. No. : 93141-87482, 0744-2209671


IVRS No : 0744-2439051, 52, 53, www. motioniitjee.com , hr@motioniitjee.com

Downloaded from www.iitjeephysics4u.com


KINEMATICS Page # 51

9. Which of the following graphs cannot possibily 12. Give an example from your own experience in which
represent one dimensional motion of a particle? the velocity of an object is zero for just an instant of
x  time, but its acceleration is not zero.
|v| l
Sol.
t t
t

x - displacement l - length of path


Sol.

13. A ball is dropped from rest from the top of a building


and strikes the ground with a speed vf. From ground
level, a second ball is thrown straight upward at the
10. Can you suggest a suitable situation from same instant that the first ball is dropped. The initial
observation around you for each of the following ? speed of the second ball is v0 = vf, the same speed
x with which the first ball will eventually strike the ground.
x
Ignoring air resistance, decide whether the balls cross
paths at half the height of the building above the
halfway point, or below the halfway point. Give your
t reasoning.
t Sol.
x - displacement
x

Sol.

14. The muzzle velocity of a gun is the velocity of the


bullet when it leaves the barrel. The muzzle velocity
of one rifle with a short barrel is greater than the
muzzle velocity of another rifle that has a longer barrel.
In which rifle is the acceleration of the bullet larger?
11. One of the following statements is incorrect. Explain your reasoning.
(a) The car traveled around the track at a constant Sol.
velocity
(b) The car traveled around the track at a constant
speed. Which statement is incorrect and why ?
Sol.

394,50 - Rajeev Gandhi Nagar Kota, Ph. No. : 93141-87482, 0744-2209671


IVRS No : 0744-2439051, 52, 53, www. motioniitjee.com , hr@motioniitjee.com

Downloaded from www.iitjeephysics4u.com


Page # 52 KINEMATICS

15. On a riverboat cruise, a plastic bottle is accidentally 18. A child is playing on the floor of a recreational
dropped overboard. A passenger on the boat estimate vehicle (RV) as it moves along the highway at a
that the boat pulls ahead of the bottle by 5 meters constant velocity. He has a toy cannon, which shoots
each second. Is it possible to conclude that the boat a marble at a fixed angle and speed with respect to
is moving at 5 m/s with respect to the shore? Account the floor. The connon can be aimed toward the front
for your answer. or the rear of the RV. Is the range towards the front
Sol. the same as, less than, or greater than the range
towards the rear? Answer this question (a) from the
child’s point of view and (b) from the point of view of
an observer standing still on the ground. Justify your
answers.
Sol.

16. A wrench is accidentally dropped from the top of


the mast on a sailboat. Will the wrench hit at the
same place on the deck whether the sailboat is at
rest or moving with a constant velocity? Justify your
answer.
Sol.
19. Three swimmers can swim equally fast relative to
the water. They have a race to see who can swim
across a river in the least time. Swimmer A swims
perpendicular to the current and lands on the far shore
downstream, because the current has swept him in
that direction. Swimmer B swims upstream at an angle
to the current and lands on the far shore directly
opposite the starting point. Swimmer C swims
downstream at an angle to the current in an attempt
to take advantage of the current. Who crosses the
river in the least time? Account for your answer.
Sol.
17. Is the acceleration of a projectile equal to zero
when it reaches the top of its trajectory? If not, why
not?
Sol.

394,50 - Rajeev Gandhi Nagar Kota, Ph. No. : 93141-87482, 0744-2209671


IVRS No : 0744-2439051, 52, 53, www. motioniitjee.com , hr@motioniitjee.com

Downloaded from www.iitjeephysics4u.com


KINEMATICS Page # 53

Exercise - I (Objective Problems)


1. A hall has the dimensions 10m × 10m × 10 m. A fly 3. A body covers first 1/3 part of its journey with a
starting at one corner ends up at a diagonally opposite velocity of 2 m/s, next 1/3 part with a velocity of 3 m/s
corner. The magnitude of its displacement is nearly and rest of the journey with a velocity 6m/s. The
average velocity of the body will be
(A) 5 3 m (B) 10 3 m (C) 20 3 m (D) 30 3 m
11 8 4
Sol. (A) 3 m/s (B) m/s (C) m/s (D) m/s
3 3 3
Sol.

4. A car runs at constant speed on a circular track of


radius 100 m taking 62.8 s on each lap. What is the
average speed and average velocity on each complete
lap ?
2. A car travels from A to B at a speed of 20 km h–1m (A) velocity 10 m/s speed 10 m/s
and returns at a speed of 30 km h–1. The average (B) velocity zero, speed 10 m/s
speed of the car for the whole journey is (C) velocity zero, speed zero
(A) 5 km h–1 (B) 24 km h–1 (C) 25 km h–1(D) 50 km h–1 (D) velocity 10 m/s, speed zero
Sol. Sol.

394,50 - Rajeev Gandhi Nagar Kota, Ph. No. : 93141-87482, 0744-2209671


IVRS No : 0744-2439051, 52, 53, www. motioniitjee.com , hr@motioniitjee.com

Downloaded from www.iitjeephysics4u.com


Page # 54 KINEMATICS

5. The displacement of a body is given by 2s = gt2 Sol.


where g is a constant. The velocity of the body at
any time t is
(A) gt (B) gt/2 (C) gt2/2 (D) gt3/3
Sol.

6. A particle has a velocity u towards east at t = 0.


Its acceleration is towards west and is constant, Let
xA and xB be the magnitude of displacements in the
first 10 seconds and the next 10 seconds.
(A) xA < xB (B) xA = xB (C) xA > xB
(D) the information is insufficient to decide the relation
of xA with xB.
Sol.

8. A stone is dropped into a well in which the level of


water is h below the top of the well. If v is velocity of
sound, the time T after which the splash is heard is
given by
2h h
(A) T = 2h/v (B) T = g
+
v
2h h h 2h
(C) T = +
g 2v (D) T = +
2g v
Sol.

7. A body starts from rest and is uniformly accelerated


for 30 s. The distance travelled in the first 10s is x1,
next 10 s is x2 and the last 10 s is x3. Then x1 : x2 : x3
is the same as
(A) 1 : 2 : 4 (B) 1 : 2 : 5
(C) 1 : 3 : 5 (D) 1 : 3 : 9

394,50 - Rajeev Gandhi Nagar Kota, Ph. No. : 93141-87482, 0744-2209671


IVRS No : 0744-2439051, 52, 53, www. motioniitjee.com , hr@motioniitjee.com

Downloaded from www.iitjeephysics4u.com


KINEMATICS Page # 55

9. The co-ordinates of a moving particle at a time t, 11. A body of mass 1 kg is acted upon by a force
are given by, x = 5 sin 10 t, y = 5 cos 10 t. The speed 
F = 2 sin 3 πt i + 3 cos 3 πt j find its position at t = 1 sec
of the particle is -
(A) 25 (B) 50 (C) 10 (D) None if at t = 0 it is at rest at origin.
Sol.  3 3   2 2 
(A)  2 ,  (B)  2 , 
 3 π 9π 2   3π 3π 2 

 2 2 
(C)  ,  (D) none of these
 3π 3π 2 
Sol.

10. A body moves with velocity v = lnx m/s where x is


its position. The net force acting on body is zero at .
(A) 0 m (B) x = e2m (C) x = em (D) x = 1 m
Sol.

394,50 - Rajeev Gandhi Nagar Kota, Ph. No. : 93141-87482, 0744-2209671


IVRS No : 0744-2439051, 52, 53, www. motioniitjee.com , hr@motioniitjee.com

Downloaded from www.iitjeephysics4u.com


Page # 56 KINEMATICS

12. A force F = Be–Ct acts on a particle whose mass is Sol.


m and whose velocity is 0 at t = 0. It’s terminal velocity
(velocity after a long time) is :
C B BC B
(A) (B) (C) (D) –
mB mC m mC
Sol.

13. A particle starts moving rectilinearly at time t = 0


such that its velocity ‘v’ changes with time ‘t’ according
to the equation v = t2 – t where t is in seconds and v
is in m/s. The time interval for which the particle retards
is
(A) t < 1/2 (B) 1/2 < t < 1
(C) t > 1 (D) t < 1/2 and t > 1
Sol.
15. A particle is projected vertically upwards from a
point A on the ground. It takes t1 time to reach a
point B but it still continues to move up. If it takes
further t2 time to reach the ground from point B then
height of point B from the ground is
1
(A) g( t1 + t 2 ) 2 (B) g t1 t2
2
1 1
(C) g( t1 + t 2 ) 2 (D) gt 1t 2
8 2
Sol.

14. A ball is thrown vertically down with velocity of


5m/s. With what velocity should another ball be thrown
down after 2 seconds so that it can hit the 1st ball in 2
seconds
(A) 40 m/s (B) 55 m/s (C) 15 m/s (D) 25 m/s

394,50 - Rajeev Gandhi Nagar Kota, Ph. No. : 93141-87482, 0744-2209671


IVRS No : 0744-2439051, 52, 53, www. motioniitjee.com , hr@motioniitjee.com

Downloaded from www.iitjeephysics4u.com


KINEMATICS Page # 57

16. Balls are thrown vertically upward in such a way 18. The displacement-time graph of a moving particle
that the next ball is thrown when the previous one is is shown below. The instantaneous velocity of the
at the maximum height. If the maximum height is 5m, particle is negative at the point
the number of balls thrown per minute will be
x
(A) 40 (B) 50 (C) 60 (D) 120
Sol. D

E F
C
t
(A) C (B) D (C) E (D) F
Sol.

17. A disc arranged in a vertical plane has two groves


19. The variation of velocity of a particle moving along
of same length directed along the vertical chord AB
straight line is shown in the figure. The distance
and CD as shown in the fig. The same particles slide
travelled by the particle in 4 s is
down along AB and CD. The ratio of the time tAB/tCD is

A C v(m/s)
60º
D
20
B 10

(A) 1 : 2 (B) 1: 2 (C) 2 : 1 (D) 2 :1 t(s)


1 2 3 4
Sol.
(A) 25m (B) 30m (C) 55m (D) 60m
Sol.

394,50 - Rajeev Gandhi Nagar Kota, Ph. No. : 93141-87482, 0744-2209671


IVRS No : 0744-2439051, 52, 53, www. motioniitjee.com , hr@motioniitjee.com

Downloaded from www.iitjeephysics4u.com


Page # 58 KINEMATICS

20. The displacement time graphs of two particles A 22. Acceleration versus velocity graph of a particle
and B are straight lines making angles of respectively
moving in a straight line starting form rest is as shown
30º and 60º with the time axis. If the velocity of A is
in figure. The corresponding velocity-time graph would
vA
vA and that of B is vB then the value of v is be -
B a
(A) 1/2 (B) 1 / 3 (C) 3 (D) 1/3
Sol.
v

v v

(A) (B)
t t
v v

21. If position time graph of a particle is sine curve as


shown, what will be its velocity-time graph (C) (D)
t t
x Sol.

v v

(A) (B)
t t

v v

(C) (D)
t t

Sol.

394,50 - Rajeev Gandhi Nagar Kota, Ph. No. : 93141-87482, 0744-2209671


IVRS No : 0744-2439051, 52, 53, www. motioniitjee.com , hr@motioniitjee.com

Downloaded from www.iitjeephysics4u.com


KINEMATICS Page # 59

23. A man moves in x - y plane along the path shown. Question No. 25 to 27 (3 questions)
At what point is his average velocity vector in the
same direction as his instantaneous velocity vector. The x-t graph of a particle moving along a straight
The man starts from point P. line is shown in figure
y
C x parabola
PB D
A
x
(A) A (B) B (C) C (D) D 0 T 2T
Sol.
25. The v-t graph of the particle is correctly shown
by

v
v
T 2T
0 T 0
(A) 2T t (B) t

v v

0 T 2T 0 T 2T
(C) t (D) t
24. The acceleration of a particle which moves along
the positive x-axis varies with its position as shown.
If the velocity of the particle is 0.8 m/s at x = 0, the Sol.
velocity of the particle at x = 1.4 is (in m/s)
2
a (in m/s )

0.4

0.2

O 0.4 0.8 1.4 x (in m)

(A) 1.6 (B) 1.2 (C) 1.4 (D) none


Sol.

26. The a-t graph of the particle is correctly shown


by
a a
2T
0 0
(A) T t (B) t

a v

0 0
(C) t (D) t

394,50 - Rajeev Gandhi Nagar Kota, Ph. No. : 93141-87482, 0744-2209671


IVRS No : 0744-2439051, 52, 53, www. motioniitjee.com , hr@motioniitjee.com

Downloaded from www.iitjeephysics4u.com


Page # 60 KINEMATICS

Sol. 28. Choose the incorrect statement. The particle comes


to rest at
(A) t = 0 s (B) t = 5 s (C) t = 8 s
(D) none of these
Sol.

27. The speed-time graph of the particle is correctly


shown by 29. Identify the region in which the rate of change of

∆v
speed speed velocity of the particle is maximum
∆t
(A) 0 (B) 0
T 2T t T 2T t (A) 0 to 2s (B) 2 to 4s (C) 4 to 6s (D) 6 to 8 s
Sol.

speed speed

(C) 0 0
T 2T t (D) T 2T t

Sol.

30. If the particle starts from the position x0 = –15 m,


then its position at t = 2s will be
(A) – 5m (B) 5m (C) 10 m (D) 15 m
Sol.

Question No. 28 to 33 (6 questions)


The figure shows a velocity-time graph of a particle
moving along a straight line

v(ms–1)
10

0
2 4 6 8 t(s)

–20

394,50 - Rajeev Gandhi Nagar Kota, Ph. No. : 93141-87482, 0744-2209671


IVRS No : 0744-2439051, 52, 53, www. motioniitjee.com , hr@motioniitjee.com

Downloaded from www.iitjeephysics4u.com


KINEMATICS Page # 61

31. The maximum of displacement of the particle is 33. The correct displacement-time graph of the particle
(A) 33.3 m (B) 23.3 m (C) 18.3 (D) zero is shown as
x x
Sol.
(m) (m)

(A) (B)
0 2 4 6 8 t(s) 0 2 4 6 8 t(s)
x x
(m) (m)

(C) (D)
0 2 4 6 8 t(s) 0 2 4 6 8 t(s)
Sol.

34. The velocity-time graph of a body falling from


rest under gravity and rebounding from a solid surface
is represented by which of the following graphs ?
V
V

(A) t (B)
t

V V

(C) (D)
t t
32. The total distance travelled by the particle is
(A) 66.6 m (B) 51.6 m (C) zero (D) 36.6 m
Sol. Sol.

394,50 - Rajeev Gandhi Nagar Kota, Ph. No. : 93141-87482, 0744-2209671


IVRS No : 0744-2439051, 52, 53, www. motioniitjee.com , hr@motioniitjee.com

Downloaded from www.iitjeephysics4u.com


Page # 62 KINEMATICS

35. Shown in the figure are the displacement time 37. A body A is thrown vertically upwards with such a
graph for two children going home from the school. velocity that it reaches a maximum height of h.
Which of the following statements about their relative Simultaneously another body B is dropped from height
motion is true after both of them started moving ? h. It strikes the ground and does not rebound. The
Their relative velocity: velocity of A relative to B v/s time graph is best
represented by : (upward direction is positive)
X
C1 VAB VAB

C2 (A) (B)
t t

O VAB VAB
t T
(A) first increases and then decreases t
(C) (D)
(B) first decreases and then increases
(C) is zero t
(D) is non zero constant Sol.
Sol.

36. Shown in the figure are the v


P1
velocity time graphs of the two
particles P1 and P2. Which of P2
the following statements about
their relative motion is true ?
Theire relative velocity
(A) is zero O t T
(B) is non-zero but constant 38. An object A is moving with 10 m/s and B is moving
(C) continuously decreases with 5 m/s in the same direction of positive x-axis. A
(D) continuously increases is 100 m behind B as shown. Find time taken by A to
Sol. Meet B
10m/s 5m/s
A B

100m
(A) 18 sec. (B) 16 sec. (C) 20 sec. (D) 17 sec.
Sol.

394,50 - Rajeev Gandhi Nagar Kota, Ph. No. : 93141-87482, 0744-2209671


IVRS No : 0744-2439051, 52, 53, www. motioniitjee.com , hr@motioniitjee.com

Downloaded from www.iitjeephysics4u.com


KINEMATICS Page # 63

39. It takes one minute for a passenger standing on 42. A point mass is projected, making an acute angle
an escalator to reach the top. If the escalator does 
with the horizontal. If angle between velocity v and
not move it takes him 3 minute to walk up. How long 
will it take for the passenger to arrive at the top if he acceleration g is θ, then θ is given by
walks up the moving escalator? (A) 0º < θ < 90º (B) θ = 90º
(A) 30 sec (B) 45 sec (C) 40 sec (D) 35 sec (C) θ = 90º (D) 0º < θ < 180º
Sol. Sol.

43. The velocity at the maximum height of a projectile


is half of its initial velocity u. Its range on the horizontal
plane is
40. A body is thrown up in a lift with a velocity u
relative to the lift and the time of flight is found to be 2u 2 3 u2 u2 u2
t. The acceleration with which the lift is moving up is (A) (B) (C) (D)
3g 2g 3g 2g
Sol.
u – gt 2u – gt u + gt 2u + gt
(A) (B) (C) (D)
t t t t
Sol.

Question No. 44 to 46
A projectile is thrown with a velocity of 50 ms–1 at an
angle of 53º with the horizontal
41. A ball is thrown upwards. It returns to ground
describing a parabolic path. Which of the following 44. Choose the incorrect statement
remains constant ? (A) It travels vertically with a velocity of 40 ms–1
(A) speed of the ball (B) It travels horizontally with a velocity of 30 ms–1
(B) kinetic energy of the ball (C) The minimum velocity of the projectile is 30 ms–1
(C) vertical component of velocity (D) None of these
(D) horizontal component of velocity. Sol.
Sol.

394,50 - Rajeev Gandhi Nagar Kota, Ph. No. : 93141-87482, 0744-2209671


IVRS No : 0744-2439051, 52, 53, www. motioniitjee.com , hr@motioniitjee.com

Downloaded from www.iitjeephysics4u.com


Page # 64 KINEMATICS

45. Determine the instants at which the projectile is 48. A ball is thrown from a point on ground at some
at the same height angle of projection. At the same time a bird starts
(A) t = 1s and t = 7s (B) t = 3s and t = 5s from a point directly above this point of projection at
(C) t = 2s and t = 6s (D) all the above a height h horizontally with speed u. Given that in its
Sol. flight ball just touches the bird at one point. Find the
distance on ground where ball strikes

h 2h 2h h
(A) 2u (B) u (C) 2u (D) u
g g g g
Sol.

46. The equation of the trajectory is given by


(A) 180y = 240 x – x2 (B) 180 y = x2 – 240x
(C) 180y = 135x – x 2
(D) 180y = x2 – 135x
Sol.

49.A ball is hit by a batsman at an angle of 37º as


shown in figure. The man standing at P should run at
what minimum velocity so that he catches the ball
before it strikes the ground. Assume that height of
man is negligible in comparison to maximum height of
projectile.

47. A particle is projected from a horizontal plane (x-


z plane) such that its velocity vector at time t is

given by V = aî + (b – ct ) ĵ . Its range on the horizontal
plane is given by (A) 3 ms–1 (B) 5 ms–1 (C) 9 ms–1 (D) 12 ms–1
ba 2ba 3 ba Sol.
(A) (B) (C) (D) None
c c c
Sol.

394,50 - Rajeev Gandhi Nagar Kota, Ph. No. : 93141-87482, 0744-2209671


IVRS No : 0744-2439051, 52, 53, www. motioniitjee.com , hr@motioniitjee.com

Downloaded from www.iitjeephysics4u.com


KINEMATICS Page # 65

50. A projectile is fired with a speed u at an angle θ Question No. 53 & 54 (2 questions)
with the horizontal. Its speed when its direction of At t = 0 a projectile is fired from a point O (taken as
motion makes an angle ‘α’ with the horizontal is - origin) on the ground with a speed of 50 m/s at an
(A) u secθ cosα (B) u secθ sinα angle of 53° with the horizontal. It just passes two
(C) u cosθ secα (D) u sinθ secα points A & B each at height 75 m above horizontal as
Sol. shown.

50m/s
A B

75m
53°
O
53. The horizontal separation between the points A
51. Two projectiles A and B are thrown with the same and B is -
speed such that A makes angle θ with the horizontal (A) 30 m (B) 60 m (C) 90 m (D) None
and B makes angle θ with the vertical, then - Sol.
(A) Both must have same time of flight
(B) Both must achieve same maximum height
(C) A must have more horizontal range than B
(D) Both may have same time of flight
Sol.

52. Suppose a player hits several baseballs. Which


baseball will be in the air for the longest time?
(A) The one with the farthest range.
(B) The one which reaches maximum height 54 The distance (in metres) of the particle from origin
(C) The one with the greatest initial velocity at t = 2 sec.
(D) The one leaving the bat at 45° with respect to
the ground. (A) 60 2 (B) 100 (C) 60 (D) 120
Sol. Sol.

394,50 - Rajeev Gandhi Nagar Kota, Ph. No. : 93141-87482, 0744-2209671


IVRS No : 0744-2439051, 52, 53, www. motioniitjee.com , hr@motioniitjee.com

Downloaded from www.iitjeephysics4u.com


Page # 66 KINEMATICS

55. Particle is dropped from the height of 20 m from 58. One stone is projected horizontally from a 20 m
horizontal ground. There is wind blowing due to which high cliff with an initial speed of 10 ms–1. A second
horizontal acceleration of the particle becomes 6 ms–2. stone is simultaneously dropped from that cliff. Which
Find the horizontal displacement of the particle till it of the following is true ?
reaches ground. (A) Both strike the ground with the same velocity
(A) 6 m (B) 10 m (C) 12 m (D) 24 m (B) The ball with initial speed 10ms–1 reaches the
Sol. ground first
(C) Both the balls hit the ground at the same time
(D) One cannot say without knowing the height of
the building
Sol.

56. A ball is projected from top of a tower with a


velocity of 5 m/s at an angle of 53º to horizontal. Its
speed when it is at a height of 0.45 m from the point
of projection is
(A) 2 m/s (B) 3 m/s
(C) 4 m/s (D) data insufficient
Sol. 59. An aeroplane flying at a constant velocity releases
a bomb. As the bomb drops down from the aeroplane.
(A) it will always be vertically below the aeroplane
(B) it will always be vertically below the aeroplane
only if the aeroplane is flying horizontally
(C) it will always be vertically below the aeroplane
only if the aeroplane is flying at an angle of 45° to the
horizontal.
(D) it will gradually fall behind the aeroplane if the
aeroplane is flying horizontally
Sol.

57. Find time of flight of projectile thrown horizontally


with speed 10 ms–1 from a long inclined plane which
makes an angle of θ = 45º from horizontal.
(A) 2 sec (B) 2 2 sec (C) 2 sec (D) none
Sol.

394,50 - Rajeev Gandhi Nagar Kota, Ph. No. : 93141-87482, 0744-2209671


IVRS No : 0744-2439051, 52, 53, www. motioniitjee.com , hr@motioniitjee.com

Downloaded from www.iitjeephysics4u.com


KINEMATICS Page # 67

60. A particle is projected at angle 37º with the incline Sol.


plane in upward direction with speed 10 m/s. The angle
of incline plane is given 53º. Then the maximum height
above the incline plane attained by the particle will be
(A) 3m (B) 4m (C) 5m (D) zero
Sol.

63. If time taken by the projectile to reach Q is T,


than PQ =

v
90° P

θ
61. On an inclined plane of inclination 30°, a ball is Q
thrown at an angle of 60° with the horizontal from the
(A) Tvsinθ (B) Tvcosθ (C) Tv secθ (D) Tv tanθ
foot of the incline with velocity of 10 3 ms–1. If g =
Sol.
10 ms–2, then the time in which ball with hit the inclined
plane is -
(A) 1.15 sec. (B) 6 sec
(C) 2 sec (D) 0.92 sec
Sol.

Question No. 64 to 67 (4 questions)


Two projectiles are thrown simultaneously in
62. A projectile is fired with a velocity at right angle the same plane from the same point. If their velocities
to the slope which is inclined at an angle θ with the are v1 and v2 at angles θ1 and θ2 respectively from the
horizontal. The expression for the range R along the horizontal, then answer the following questions.
incline is - 64. The trajectory of particle 1 with respect to particle
2v 2 2v 2 2 will be
(A) sec θ (B) tan θ (A) a parabola
g g
(B) a straight line
2v 2 v2 (C) a vertical straight line
(C) tan θ sec θ (D) tan2 θ
g g (D) a horizontal straight line

394,50 - Rajeev Gandhi Nagar Kota, Ph. No. : 93141-87482, 0744-2209671


IVRS No : 0744-2439051, 52, 53, www. motioniitjee.com , hr@motioniitjee.com

Downloaded from www.iitjeephysics4u.com


Page # 68 KINEMATICS

Sol. Sol.

65. If v1 cosθ1 = v2 cosθ2, then choose the incorrect


statement
(A) one particle will remain exactly below or above
the other particle
(B) the trajectory of one with respect to other will be
a vertical straight line
(C) both will have the same range
(D) none of these
Sol.
68. A helicopter is flying south with a speed of 50
kmh–1. A train is moving with the same speed towards
east. The relative velocity of the helicopter as seen
by the passengers in the train will be towards.
(A) north east (B) south east
(C) north west (D) south west
Sol.

66. If v1sinθ1 = v2sinθ2, then choose the incorrect


statement
(A) the time of flight of both the particles will be same
(B) the maximum height attained by the particles will
be same
(C) the trajectory of one with respect to another will
be a horizontal straight line 69. Two particles are moving with velocities v1 and
(D) none of these v2. Their relative velocity is the maximum, when the
Sol. angle between their velocities is
(A) zero (B) π/4 (C) π/2 (D) π
Sol.

67. If v1 = v2 and θ1 > θ2, then choose the incorrect


statement
(A) Particle 2 moves under the particle 1
(B) The slope of the trajectory of particle 2 with
respect to 1 is always positive
(C) Both the particle will have the same range if θ1 >
45° and θ2 < 45° and θ1 + θ2 = 90°
(D) none of these

394,50 - Rajeev Gandhi Nagar Kota, Ph. No. : 93141-87482, 0744-2209671


IVRS No : 0744-2439051, 52, 53, www. motioniitjee.com , hr@motioniitjee.com

Downloaded from www.iitjeephysics4u.com


KINEMATICS Page # 69

70. A ship is travelling due east at 10 km/h. A ship 73. A swimmer’s speed in the direction of flow of river
heading 30º east of north is always due north from is 16 km h–1. Against the direction of flow of river, the
the first ship. The speed of the second ship in km/h is swimmer’s speed is 8 km h–1. Calculate the swimmer’s
(A) 20 2 (B) 20 3 / 2 (C) 20 (D) 20 / 2 speed in still water and the velocity of flow of the
Sol. river.
(A) 12 km/h, 4 km/h (B) 10 km/h, 3 km/h
(C) 10 km/h, 4 km/h (D) 12 km/h, 2 km/h
Sol.

71. A particle is kept at rest at origin. Another particle


starts from (5, 0) with a velocity of – 4 î + 3 ĵ . Find
their closest distance of approach.
(A) 3 m (B) 4 m (C) 5 m (D) 2 m
Sol.

74. A pipe which can rotate in a vertical plane is


mounted on a cart. The cart moves uniformly along a
horizontal path with a speed v1 = 2 m/s. At what
angle α to the horizontal should the pipe be placed so
that drops of rain falling with a velocity v2 6 m/s move
parallel to the walls of the pipe without touching them ?
consider the velocity of the drops as constant due to
the resistance of the air.

v1
72. Four particles situated at the corners of a square
of side ‘a’ move at a constant speed v. Each particle
maintains a direction towards the next particle in
–1  1 
succession. Calculate the time particles will take to (A) tan –1( 3) (B) tan  
3
meet each other.
a a a 2a –1  1 
(A) (B) (C) (D) (C) tan   (D) None of these
v 2v 3v 3v 2
Sol. Sol.

394,50 - Rajeev Gandhi Nagar Kota, Ph. No. : 93141-87482, 0744-2209671


IVRS No : 0744-2439051, 52, 53, www. motioniitjee.com , hr@motioniitjee.com

Downloaded from www.iitjeephysics4u.com


Page # 70 KINEMATICS

77. A flag is mounted on a car moving due North with


velocity of 20 km/hr. Strong winds are blowing due
East with velocity of 20 km/hr. The flag will point it
direction
(A) East (B) North-East
(C) South-East (D) South-West
Sol.

75. A swimmer swims in still water at a speed = 5 km/


hr. He enters a 200 m wide river, having river flow speed
= 4 km/hr at point A and proceeds to swim at an angle
of 127° (sin37° = 0.6) with the river flow direction.
Another point B is located directly across A on the
other side. The swimmer lands on the other bank at a
point C, from which he walks the distance CB with a
speed = 3 km/hr. The total time in which he reachrs
from A to B is
(A) 5 minutes (B) 4 minutes
(C) 3 minutes (D) None
Sol.
78. A man is crossing a river flowing with velocity of 5
m/s. He reaches a point directly across at a distance
of 60 m in 5 sec. His velocity in still water should be
(A) 12 m/s (B) 13 m/s (C) 5 m/s (D) 10 m/s
Sol.

79. Wind is blowing in the north direction at speed of


2 m/s which causes the rain to fall at some angle with
76. A boat having a speed of 5 km/hr. in still water, the vertical. With what velocity should a cyclist drive
crosses a river of width 1 km along the shortest possible so that the rain appears vertical to him
path in 15 minutes. The speed of the river in Km/hr. (A) 2 m/s south (B) 2 m/s north
(A) 1 (B) 3 (C) 4 (D) 41 (C) 4 m/s west (D) 4 m/s south
Sol.

394,50 - Rajeev Gandhi Nagar Kota, Ph. No. : 93141-87482, 0744-2209671


IVRS No : 0744-2439051, 52, 53, www. motioniitjee.com , hr@motioniitjee.com

Downloaded from www.iitjeephysics4u.com


KINEMATICS Page # 71

Exercise - II (One or more than one option is correct)

1. The displacement x of a particle depend on time t Sol.


as x = αt2 – β t3
(A) particle will return to its starting point after time α/β.

(B) the particle will come to rest after time

(C) the initial velocity of the particle was zero but its
initial acceleration was not zero.
α
(D) no net force act on the particle at time

Sol.

3. Mark the correct statements for a particle going on


a straight line
(A) if the veloci ty is zero at any instant, the
acceleration should also be zero at that instant
(B) if the velocity is zero for a time interval, the
acceleration is zero at any instant within the time
interval
(C) if the velocity and acceleration have opposite sign,
the object is slowing down
(D) if the position and velocity have opposite sign,
the particle is moving towards the origin.
Sol.

4. A particle initially at rest is subjected to two forces.


One is constant, the other is a retarding force
proportion at to the particle velocity. In the subsequent
motion of the particle.
2. A particle has intial velocity 10 m/s. It moves due
(A) the acceleration will increase from zero to a
to constant retarding force along the line of velocity
constant value
which produces a retardation of 5 m/s2. Then -
(B) the acceleration will decrease from its initial value
(A) the maximum displacement in the direction of initial
to zero
velocity is 10 m
(C) the velocity will increase from zero to maximum &
(B) the distance travelled in first 3 seconds is 7.5 m
then decrease
(C) the distance travelled in first 3 seconds is 12.5 m
(D) the velocity will increase from zero to a constant
(D) the distance travelled in first 3 seconds is 17.5 m
value.

394,50 - Rajeev Gandhi Nagar Kota, Ph. No. : 93141-87482, 0744-2209671


IVRS No : 0744-2439051, 52, 53, www. motioniitjee.com , hr@motioniitjee.com

Downloaded from www.iitjeephysics4u.com


Page # 72 KINEMATICS

Sol. Sol.

 
5. Let v and a denote the velocity and acceleration
respectively of a body in one-dimensional motion
  7. Let v and a denote the velocity and acceleration
(A) | v| must decrease when a < 0 respectively of a body
 (A) a can be non zero when v = 0
(B) Speed must increase when a > 0
  (B) a must be zero when v = 0
(C) Speed will increase when both v and a are < 0 (C) a may be zero when v ≠ 0
 
(D) Speed will decrease when v < 0 and a > 0 (D) The direction of a must have some correlation
Sol. with the direction of v
Sol.

6. Which of the following statements are true for a 8. A bead is free to slide down a A
moving body? sm ooth wi re ti g ht l y st ret ched
(A) If its speed changes, its velocity must change between points A and B on a vertical θ
B R
and it must have some acceleration circle. If the bead starts from rest
(B) If its velocity changes, its speed must change at A, the highest point on the circle
and it must have some acceleration (A) its velocity v on arriving at B is proportional to
(C) If its velocity changes, its speed may or may not cosθ
change, and it must have some acceleration (B) its velocity v on arriving B is proportional to tanθ
(D) If its speed changes but direction of motion does (C) time to arrive at B is proportional to cosθ
not changes, its velocity may remain constant (D) time to arrive at B is independent of θ

394,50 - Rajeev Gandhi Nagar Kota, Ph. No. : 93141-87482, 0744-2209671


IVRS No : 0744-2439051, 52, 53, www. motioniitjee.com , hr@motioniitjee.com

Downloaded from www.iitjeephysics4u.com


KINEMATICS Page # 73

Sol. 10. The figure shows the velocity (v) of a particle


plotted against time (t)

+v0
v
T
O
t 2T
–v0

(A) The particle changes its direction of motion at


some point
(B) The acceleration of the particle remains constant
(C) The displacement of the particle is zero
(D) The initial and final speeds of the particle are the
same
Sol.

9. Velocity-time graph for a car is semicircle as shown


here. Which of the following is correct :
v
1m/s

2 sec
(A) Car must move in circular path
(B) Acceleration of car is never zero 11. A block is thrown with a velocity of 2 ms–1 (relative
(C) Mean speed of the particle is π/4 m/s. to ground) on a belt, which is moving with velocity 4
(D) The car makes a turn once during its motion ms–1 in opposite direction of the initial velocity of block.
If the block stops slipping on the belt after 4 sec of
Sol.
the throwing then choose the correct statements(s)
(A) Displacement with respect to ground is zero after
2.66 sec and magnitude of displacement with respect
to ground is 12 m after 4 sec.
(B) Magnitude of displacement with respect to ground
in 4 sec is 4 m.
(C) Magnitude of displacement with respect to belt in
4 sec is 12 m.
(D) Displacement with respect to ground is zero in 8/
3 sec.
Sol.

394,50 - Rajeev Gandhi Nagar Kota, Ph. No. : 93141-87482, 0744-2209671


IVRS No : 0744-2439051, 52, 53, www. motioniitjee.com , hr@motioniitjee.com

Downloaded from www.iitjeephysics4u.com


Page # 74 KINEMATICS

13. An observer moves with a constant speed along


the line joining two stationary objects. He will observe
that the two objects
(A) have the same speed
(B) have the same velocity
(C) move in the same direction
(D) move in opposite directions
Sol.

14. A man on a rectilinearly moving cart, facing the


direction of motion, throws a ball straight up with
respect to himself
(A) The ball will always return to him
(B) The ball will never return to him
(C) The ball will return to him if the cart moves with
constant velocity
(D) The ball will fall behind him if the cart moves with
12. A particle moves with constant speed v along a some acceleration
regular hexagon ABCDEF in the same order. Then the Sol.
magnitude of the average velocity for its motion from
A to -
(A) F is v/5 (B) D is v/3
(C) C is v √3/2 (D) B is v
Sol.

15. A projectile of mass 1 kg is projected with a velocity


of 20 m/s such that it strikes on the same level as
the point of projection at a distance of3 m. Which
of the following options are incorrect.
(A) the maximum height reached by the projectile can
be 0.25 m
(B) the minimum velocity during its motion can be
15 m/s
3
(C) the time taken for the flight can be sec.
5
(D) maximum potential energy during its motion can
be 6J.

394,50 - Rajeev Gandhi Nagar Kota, Ph. No. : 93141-87482, 0744-2209671


IVRS No : 0744-2439051, 52, 53, www. motioniitjee.com , hr@motioniitjee.com

Downloaded from www.iitjeephysics4u.com


KINEMATICS Page # 75

Sol. 17. If T is the total time of flight, h is the maximum


height & R is the range for horizontal motion, the x & y
co-ordinates of projectile motion and time t are related
as :

 t t  X  X
(A) y = 4h   1 −  (B) y = 4h   1 − 
 T  T  R  R

 T  T  R  R
(C) y = 4h   1 −  (D) y = 4h   1 − 
 t t  X  X
Sol.

16. Choose the correct alternative (s)


(A) If the greatest height to which a man can throw a
stone is h, then the greatest horizontal distance upto
which he can throw the stone is 2h.
(B) The angle of projection for a projectile motion whose
range R is n times the maximum height is tan–1(4/n)
(C) The time of flight T and the horizontal range R of
a projectile are connected by the equation gT2 =
2Rtanθ where θ is the angle of projection.
(D) A ball is thrown vertically up. Another ball is thrown 18. A particle moves in the xy plane with a constant
at an angle θ with the vertical. Both of them remain in acceleration ‘g’ in the negative y-direction. Its equation
air for the same period of time. Then the ratio of of motion is y = ax – bx2, where a and b constants.
heights attained by the two ball 1 : 1.
Which of the following are correct?
Sol.
(A) The x-component of its velocity is constant.
(B) At the origin, the y-component of its velocity is
g
a
2b
(C) At the origin, its velocity makes an angle tan–1(a)
with the x-axis
(D) The particle moves exactly like a projectile.
Sol.

394,50 - Rajeev Gandhi Nagar Kota, Ph. No. : 93141-87482, 0744-2209671


IVRS No : 0744-2439051, 52, 53, www. motioniitjee.com , hr@motioniitjee.com

Downloaded from www.iitjeephysics4u.com


Page # 76 KINEMATICS

20. A particle is projected from the ground with velocity


u at angle θ with horizontal. The horizontal range,
maximum height and time of flight are R, H and T
respectively. They are given by,

u 2 sin 2θ u 2 sin2 θ 2u sin θ


R= ,H= and T =
g 2g g

Now keeping u as fixed, θ is varied from 30° to 60°.


Then,
(A) R will first increase then decrease, H will increase
and T will decrease
(B) R will first increase then decrease while H and T
both will increase
(C) R will decrease while H and T will increase
(D) R will increase while H and T will increase
Sol.
19. A ball is rolled off along the edge of a horizontal
table with velocity 4 m/s. It hits the ground after time
0.4s. Which of the following are correct?
(A) The height of the table is 0.8 m
(B) It hits the ground at an angle of 60° with the
vertical
(C) It covers a horizontal distance 1.6 m from the
table
(D) It hits the ground with vertical velocity 4 m/s
Sol.

394,50 - Rajeev Gandhi Nagar Kota, Ph. No. : 93141-87482, 0744-2209671


IVRS No : 0744-2439051, 52, 53, www. motioniitjee.com , hr@motioniitjee.com

Downloaded from www.iitjeephysics4u.com


KINEMATICS Page # 77

Exercise - III (Subjective Problems)

1. The position vector of a particle moving in x-y



plane is given by r = ( t 2 − 4)i + ( t − 4)j . Find
(a) Equation of trajectory of the particle
Sol.

3. At time t the position vector of a particle of mass



m = 3kg is given by r = 6 t i − t 3 j + cos t k . Find the re-

sultant force F ( t) , magnitude of its acceleration when

π
t= , & speed when t = π.
2
(b) Time when it crosses x-axis and y-axis
Sol. Sol.

2. A p arti cl e move s al ong the sp ac e curv e



r = ( t 2 + t) i + ( 3 t − 2) j + (2t 3 − 4 t 2 ) k . (t in sec, r in m) Find
at time t = 2 the (a) velocity, (b) acceleration, (c)
speed or magnitude of velocity and (d) magnitude of
acceleration.
Sol.

4. The velocity time graph of a body moving in a


straight line is shown. Find its
y
velocity in m/sec

60°

30°
x
2.5 sec
time in sec 2

394,50 - Rajeev Gandhi Nagar Kota, Ph. No. : 93141-87482, 0744-2209671


IVRS No : 0744-2439051, 52, 53, www. motioniitjee.com , hr@motioniitjee.com

Downloaded from www.iitjeephysics4u.com


Page # 78 KINEMATICS

(a) instantaneous velocity at t = 1.5 sec


Sol.

(b) average acceleration from t = 1.5 sec. to


t = 2.5 sec 6. Velocity of car v is given by v = at – bt2, where a
Sol. and b are positive constants & t is time elapsed. Find
value of time for which velocity is maximum & also
corresponding value of velocity.
Sol.

(c) draw its acceleration time graph from t = 0 to


t = 2.5 sec
Sol.

5. The curvilinear motion of a particle is defined by vx


= 50 – 16t and y = 100 – 4t2 , where vx is in metres
per second, y is in metres and t is in seconds. It is 7. The force acting on a body moving in a straight line
also known that x = 0 at t = 0. Determine the velocity is given by F = (3t2 – 4t + 1) Newton where t is in
(v) and acceleration (a) when the position y = 0 is sec. If mass of the body is 1kg and initially it was at
reached. rest at origin. Find
Sol.
(a) displacement between time t = 0 and t = 2 sec
Sol.

394,50 - Rajeev Gandhi Nagar Kota, Ph. No. : 93141-87482, 0744-2209671


IVRS No : 0744-2439051, 52, 53, www. motioniitjee.com , hr@motioniitjee.com

Downloaded from www.iitjeephysics4u.com


KINEMATICS Page # 79

9. A particle is moving along x-axis. Initially it is located


5 m left of origin and it is moving away from the origin
and slowing down. In this coordinate system, the signs
of the initial velocity and acceleration, are
+ y
– + v0 a
(b) distance travelled between time t = 0 and t = 2
(0, 0) x
sec
Sol. –
Sol.

10. Find the change in velocity of the tip of the minute


hand (radius = 10 cm) of a clock in 45 minutes.
8. A particle goes from A to B with a speed of 40 km/ Sol.
h and B to C with a speed of 60 km/h. If AB = 6BC the
ave rage speed i n k m/h betwe en A and C i s
____________
total dis tan ce travelled
[Hint : Average speed = ]
time taken
Sol.

394,50 - Rajeev Gandhi Nagar Kota, Ph. No. : 93141-87482, 0744-2209671


IVRS No : 0744-2439051, 52, 53, www. motioniitjee.com , hr@motioniitjee.com

Downloaded from www.iitjeephysics4u.com


Page # 80 KINEMATICS

11. At a distance L = 400 m from the traffic light


brakes are applied to a locomotive moving at a velocity
v= 54 km/hr. Determine the position of the locomotive
relative to the traffic light 1 min after the application
of the breaks if its acceleration is –0.3 m/sec2.
Sol.

13. A car is moving along a straight line. It is taken


from rest to a velocity of 20 ms–1 by a constant
12. A train starts from rest and moves with a constant acceleration of 5ms–2. It maintains a constant velocity
acceleration of 2.0 m/s2 for half a minute. The brakes of 20 ms–1 for 5 seconds and then is brought to rest
are then applied and the train comes to rest in one again by a constant acceleration of –2 ms–2. Draw a
minute. Find velocity-time graph and find the distance covered by
(a) the total distance moved by the train, the car.
(b) the maximum speed attained by the train and Sol.
(c) the position (s) of the train at half the maximum
speed.
Sol.

394,50 - Rajeev Gandhi Nagar Kota, Ph. No. : 93141-87482, 0744-2209671


IVRS No : 0744-2439051, 52, 53, www. motioniitjee.com , hr@motioniitjee.com

Downloaded from www.iitjeephysics4u.com


KINEMATICS Page # 81

16. From the velocity-time plot shown in figure, find


the distance travelled by the particle during the first
40 seconds. Also find the average velocity during this
period.
V
14. A stone is dropped from a height h. Simultaneously 5m/s
another stone is thrown up from the ground with such t(s)
0 20
a velocity that it can reach a height of 4 h. Find the 40
–5m/s
time when two stones cross each other.
Sol. Sol.

17. The velocity-time graph of the particle moving


15. A bal loon is ascending vertical ly with an along a straight line is shown. The rate of acceleration
acceleration of 0.2 m/s2 Two stones are dropped from and deceleration is constant and it is equal to 5 ms–2.
it at an interval of 2 sec. Find the distance between If the average velocity during the motion is 20ms–1,
them 1.5 sec after the second stone is released (use then find the value of t.
g = 9.8 m/s2).
Sol.

o t 25 sec
Sol.

394,50 - Rajeev Gandhi Nagar Kota, Ph. No. : 93141-87482, 0744-2209671


IVRS No : 0744-2439051, 52, 53, www. motioniitjee.com , hr@motioniitjee.com

Downloaded from www.iitjeephysics4u.com


Page # 82 KINEMATICS

20. A particle is projected upwards with a velocity of


100 m/sec at an angle of 60º with the vertical. Find
the time when the particle will move perpendicular to
18. The fig. shows the v-t graph of a particle moving
its initial direction, taking g = 10 m/sec2.
in straight line. Find the time when particle returns to
Sol.
the starting point.
v
20

10

10 20 25 t
Sol.

gx2
21. The equation of a projectile is y = 3 x − . The
2
angle of projectile is ________ and initial velocity is
_______.
Sol.

19. A particle is projected in the X-Y plane. 2 sec


after projection the velocity of the particle makes an
angle 45º with the X-axis. 4 sec after projection, it
moves horizontally. Find the velocity of projection (use
g = 10 ms–2).
Sol.

394,50 - Rajeev Gandhi Nagar Kota, Ph. No. : 93141-87482, 0744-2209671


IVRS No : 0744-2439051, 52, 53, www. motioniitjee.com , hr@motioniitjee.com

Downloaded from www.iitjeephysics4u.com


KINEMATICS Page # 83

22. A ball is projected at an angle of 30º above with


the horizontal from the top of a tower and strikes the
ground in 5 sec at an angle of 45º with the horizontal.
Find the height of the tower and the speed with which
it was projected. [g =10 m/s2]
Sol.

24. A ball is thrown horizontally from a cliff such that


23. A rocket is launched at an angle 53º to the
it strikes ground after 5 sec. The line of sight from the
horizontal with an initial speed of 100 ms–1. It moves
point of projection to the point of hitting makes an
along its initial line of motion with an acceleration of
angle of 37º with the horizontal. What is the initial
30 ms–2 for 3 seconds. At this time its engine falls &
velocity of projection.
the rocket proceeds like a free body. Find :
(i) the maximum altitude reached by the rocket
37º
(ii) total time of flight
(iii) the horizontal range. [sin 53º = 4/5]
Sol.
Sol.

394,50 - Rajeev Gandhi Nagar Kota, Ph. No. : 93141-87482, 0744-2209671


IVRS No : 0744-2439051, 52, 53, www. motioniitjee.com , hr@motioniitjee.com

Downloaded from www.iitjeephysics4u.com


Page # 84 KINEMATICS

25. A ball is projected on smooth inclined plane in 27. The horizontal range of a projectiles is R and the
direction perpendicular to line of greatest slope with maximum height attained by it is H. A strong wind now
velocity of 8m/s. Find it’s speed after 1 sec. begins to blow in the direction of motion of the
projectile, giving it a constant horizontal acceleration
= g/2. Under the same conditions of projection, find
8 m/s
the horizontal range of the projectile.
37º Sol.

Sol.

26. Find range of projectile on the inclined plane which


is projected perpendicular to the incline plane with
velocity 20m/s as shown in figure. 28. A butterfly is flying with velocity 10 i + 12j m / s
-1
u = 20 ms and wind is blowing along x axis with velocity u. If
butterfly starts motion from A and after some time
reaches point B, find the value of u.
y
37º
B

Sol.

A 37°
x
Sol.

394,50 - Rajeev Gandhi Nagar Kota, Ph. No. : 93141-87482, 0744-2209671


IVRS No : 0744-2439051, 52, 53, www. motioniitjee.com , hr@motioniitjee.com

Downloaded from www.iitjeephysics4u.com


KINEMATICS Page # 85

29. In the figure shown, the two projectiles are fired


simultaneously. What should be the initial speed of
the left side projectile for the two projectile to hit in
mid-air ?

20m/s
60º 45º
\\\\\\\\\\\\\\\\\\\\\\\\\\
10m
Sol.

31. Two particles are moving along two long straight


lines, in the same plane, with the same speed = 20
cm/s. The angle between the two lines is 60°, and
their intersection point is O. At a certain moment, the
two particles are located at distance 3m and 4m from
O, and are moving towards O. Find the shortest
distance between them subsequently?
30. In the figure shown, the two projectiles are fired
simultaneously. Find the minimum distance between
Sol.
them during their flight?
20 3 m / s
20 m/s

60° 30°
20 m
Sol.

394,50 - Rajeev Gandhi Nagar Kota, Ph. No. : 93141-87482, 0744-2209671


IVRS No : 0744-2439051, 52, 53, www. motioniitjee.com , hr@motioniitjee.com

Downloaded from www.iitjeephysics4u.com


Page # 86 KINEMATICS

34. A man with some passengers in his boat, starts


32. Rain is falling vertically with a speed of 20 ms–1 perpendicular to flow of river 200m wide and flowing
relative to air. A person is running in the rain with a with 2m/s. Boat speed in still water is 4m/s. When he
velocity of 5 ms–1 and a wind is also blowing with a reaches half the width of river the passengers asked
speed of 15 ms–1 (both towards east). Find the angle him they want to reach the just opposite end from
with the vertical at which the person should hold his where they have started.
umbrella so that he may not get drenched. (a) Find the direction due to which he must row to
Sol. reach the required end.
(b) How many times more total time, it would take to
that if he would have denied the passengers.
Sol.

33. A glass wind screen whose inclination with the


vertical can be changed is mounted on a car. The car
moves horizontally with a speed of 2 m/s. At what
angle α with the vertical should the wind screen be
placed so that the rain drops falling vertically
downwards with velocity 6 m/s strike the wind screen
perpendicularly? 35. A man crosses a river in a boat. If he crosses the
Sol. river in minimum time he takes 10 minutes with a drift
120 m. If he crosses the river taking shortest path,
he takes 12.5 minute, find -
(i) width of the river
(ii) velocity of the boat with respect to water
(iii) speed of the current.
Assume vb/r > vr
Sol.

394,50 - Rajeev Gandhi Nagar Kota, Ph. No. : 93141-87482, 0744-2209671


IVRS No : 0744-2439051, 52, 53, www. motioniitjee.com , hr@motioniitjee.com

Downloaded from www.iitjeephysics4u.com


KINEMATICS Page # 87

Exercise - IV (Tough Subjective Problems)


1. A speeder in an automobile passes a stationary 3. The speed of a particle when it is at its greatest
policeman who is hiding behind a bill board with a
height is2 / 5 times of its speed when it is at its half
motorcycle. After a 2.0 sec delay (reaction time) the
policeman acceleraties to his maximum speed of 150 the maximum height. The angle of projection is
km/hr in 12 sec and catches the speeder 1.5 km beyond _________ and the velocity vector angle at half the
the billboard. Find the speed of speeder in km/hr. maximum height is _________.
Sol. Sol.

2. A large number of bullets are fired in all direction


with the same speed v. What is the maximum area on 4. A projectile is to be thrown horizontally from the
ground on which these bullets can spread? top of a wall of height 1.7m. Calculate the initial
Sol. velocity of projection if it hits perpendicularly an incline
of angle 37° which starts from the ground at the bottom
of the wall. The line of greatest slope of incline lies in
the plane of motion of projectile.
Sol.

394,50 - Rajeev Gandhi Nagar Kota, Ph. No. : 93141-87482, 0744-2209671


IVRS No : 0744-2439051, 52, 53, www. motioniitjee.com , hr@motioniitjee.com

Downloaded from www.iitjeephysics4u.com


Page # 88 KINEMATICS

5. Two inclined planes OA and OB having inclination 6. A particle is thrown horizontally with relative velocity
(with horizontal) 30° and 60° respectively, intersect 10 m/s from an inclined plane, which is also moving
each other at O as shown in figure. A particle is with acceleration 10 m/s2 vertically upward. Find the
time after which it lands on the plane (g = 10 m/s2)
projected from point P with velocity u = 10 3 ms –1 along
a direction perpendicular to plane OA. If the particle 2
10 m/s
strikes plane OB perpendicularly at Q, calculate
A
u B 30°
Q
P Sol.
h
30° 60°
O
(a) velocity with which particle strikes the plane OB,
(b) time of flight,
(c) vertical height h of P from O,
(d) maximum height from O attained by the particle
and
(e) distance PQ
Sol.

7. A, B & C are three objects each moving with constant



velocity. A’s speed is 10 m/sec in a direction PQ . The
velocity of B relative to A is 6 m/sec at an angle of,
cos–1(15/24) to PQ. The velocity of C relative to B is

12 m/sec in a direction QP , then find the magnitude
of the velocity of C.
Sol.

394,50 - Rajeev Gandhi Nagar Kota, Ph. No. : 93141-87482, 0744-2209671


IVRS No : 0744-2439051, 52, 53, www. motioniitjee.com , hr@motioniitjee.com

Downloaded from www.iitjeephysics4u.com


KINEMATICS Page # 89

8. A particle is projected from point P with velocity


5 2 m/s perpendicular to the surface of a hollow
right angle cone whose axis is vertical. It collides at Q
normally. Find the time of the flight of the particle.
y

P Q
45° x

Sol.

10. A hunter is riding an elephant of height 4m moving


in straight line with uniform speed of 2m/sec. A deer
running with a speed V in front at a distance of 4 5 m
moving perpendicular to the direction of motion of the
elephant. If hunter can throw his spear with a speed
of 10 m/sec. relative to the elephant, then at what
angle θ to it’s direction of motion must he throw his
spear horizontally for a successful hit. Find also the
speed ‘V’ of the deer.
Sol.

9. A glass wind screen whose inclination with the


vertical can be changed, is mounted on a cart as
shown in figure. The cart moves uniformly along the
horizontal path with a speed of 6 m/s. At what maximum
angle α to the vertical can the wind screen be placed
so that the rain drops falling vertically downwards
with velocity 2 m/s, do not enter the cart?

v=6m/s

Sol.

394,50 - Rajeev Gandhi Nagar Kota, Ph. No. : 93141-87482, 0744-2209671


IVRS No : 0744-2439051, 52, 53, www. motioniitjee.com , hr@motioniitjee.com

Downloaded from www.iitjeephysics4u.com


Page # 90 KINEMATICS

Exercise - V JEE-Problems
1. Two guns, situated at the top of a hill of height 10
m, fire one shot each with the same speed 5 3 m/s
at some interval of time. One gun fires horizontally
and other fires upwards at an angle of 60° with the
horizontal. The shots collide in air at a point P. Find
(a) the time interval between the firings, and
(b) the coordinates of the point P. Take origin of the
coordinates system at the foot of the hill right below
the muzzle and trajectories in X-Y plane.[JEE’ 1996]
Sol.

3. A large heavy box is sliding without friction down a


smooth plane of inclination θ. From a point P on the
bottom of a box, a particle is projected inside the
box. The initial speed of the particle with respect to
box is u and the direction of projection makes an angle
α with the bottom as shown in figure.

α
P Q

(a) Find the distance along the bottom of the box


between the point of projection P and the point Q
where the particle lands. (Assume that the particle
does not hit any other surface of the box. Neglect air
2. The trajectory of a projectile in a vertical plane is resistance).
y = ax – bx2, where a, b are constants & x and y are (b) If the horizontal displacement of the particle as
respectively the horizontal & vertical distances of the seen by an observer on the ground is zero, find the
projectile from the point of projection. The maximum speed of the box with respect to the ground at the
height attained is ___________ & the angle of instant when the particle was projected.
projection from the horizontal is ______, [JEE’ 1997] [JEE’ 1998]
Sol. Sol.

394,50 - Rajeev Gandhi Nagar Kota, Ph. No. : 93141-87482, 0744-2209671


IVRS No : 0744-2439051, 52, 53, www. motioniitjee.com , hr@motioniitjee.com

Downloaded from www.iitjeephysics4u.com


KINEMATICS Page # 91

5. A ball is dropped vertically from a height d above


the ground it hits the ground and bounces up vertically
to a height d/2. Neglecting subsequent motion and air
resistances, its velocity v varies with the height h
above the ground as - [JEE’ 2000 (Scr)]
v v

d
h h
(A) (B) d

v v

d d
(C) h (D) h

Sol.

4. In 1.0 sec. a particle goes from point A to point B


moving in a semicircle of radius 1.0 m. The magnitude
of average velocity is - [JEE ‘99] 6. An object A is kept fixed at the point x = 3 m and
A
y = 1.25 m on a plank P raised above the ground. At
time t = 0 the plank starts moving along the +x direction
1m with an acceleration 1.5 m/s2. At the same instant a
B stone is projected from the origin with a velocity u as
(A) 3.14 m/sec (B) 2.0 m/sec shown. A stationary person on the ground observes
(C) 1.0 m/sec (D) zero the stone hitting the object during its downward
Sol. motion at an angle of 45° to the horizontal. All the
motions are in x-y plane. Find u and the time after
which the stone hits the object. Take g = 10 m/s2
[JEE 2000]
y
A
1.25m P

u
O 3.0 m x
Sol.

394,50 - Rajeev Gandhi Nagar Kota, Ph. No. : 93141-87482, 0744-2209671


IVRS No : 0744-2439051, 52, 53, www. motioniitjee.com , hr@motioniitjee.com

Downloaded from www.iitjeephysics4u.com


Page # 92 KINEMATICS

7. On a frictionless horizontal surface, assumed to be


the x-y plane, a small trolley A is moving along a
straight line parallel to the y-axis (see figure) with a
constant velocity of ( 3 – 1) m/s. At a particular
instant, when the line OA makes an angle of 45° with 8. A particle starts from rest. Its acceleration (a)
the x-axis, a ball is thrown along the surface from the varsus time (t) is as shown in the figure. The maximum
origin O. Its velocity makes an angle φ with the x-axis speed of the particle will be - [JEE’ 2004 (Scr)]
and it hits the trolley.
y a
2
A 10m/s

45° 11 t(s)
O x
(a) The motion of the ball is observed from the frame (A) 110 m/s (B) 55 m/s (C) 550 m/s (D) 660 m/s
of trolley. Calculate the angle θ made by the velocity Sol.
vector of the ball with the x-axis in this frame.
(b) Find the speed of the ball with respect to the

surface, if φ = . [JEE 2002]
3
Sol.

394,50 - Rajeev Gandhi Nagar Kota, Ph. No. : 93141-87482, 0744-2209671


IVRS No : 0744-2439051, 52, 53, www. motioniitjee.com , hr@motioniitjee.com

Downloaded from www.iitjeephysics4u.com


KINEMATICS Page # 93

9. A small block slides without friction down an inclined 11. STATEMENT-1


plane starting from rest. Let Sn be the distance For an observer looking out through the window of a
Sn fast moving train, the nearby objects appear to move
travelled from time t = n – 1 to t = n. The S is - in the opposite direction to the train, while the distant
n+1
[JEE’ 2004 (Scr)] objects appear to be stationary.
STATEMENT-2
2n – 1 2n + 1 2n – 1 2n If the observer and the object are moving at velocities
(A) (B) (C) (D)  
2n 2n – 1 2n + 1 2n + 1
V1 and V2 respectively with reference to a laboratory
Sol. frame, the velocity of the object with respect to the
 
observer is V2 – V1
(A) STATEMENT-1 is True, STATEMENT-2 is True;
STATEMENT-2 is a correct explanation for STATEMENT-1
(B) STATEMENT-1 is True, STATEMENT-2 is True’
STATEMENT-2 is NOT a correct explanation for
STATEMENT-1
(C) STATEMENT-1 is True, STATEMENT-2 is False
(D) STATEMENT-1 is False, STATEMENT-2 is True
[JEE’ 2008]
10. The velocity displacement graph of a particle Sol.
moving along a straight line is shown. The most suitable
acceleration-displacement graph will be -
[JEE’ 2005 (Scr)]

v
v0

x0 x
a a
x

(A) (B)
x
a 12. A train is moving along a straight line with a con-
a
x x stant acceleration 'a'. A boy standing in the train
throws a ball forward with a speed of 10 m/s, at an
(C) (D) angle of 60° to the horizontal. The boy has to move
forward by 1.15 m inside the train to catch the ball
back at the initial height. The acceleration of the train
Sol. in m/s2 is [JEE’ 2011]
Sol.

394,50 - Rajeev Gandhi Nagar Kota, Ph. No. : 93141-87482, 0744-2209671


IVRS No : 0744-2439051, 52, 53, www. motioniitjee.com , hr@motioniitjee.com

Downloaded from www.iitjeephysics4u.com


Page # 94 KINEMATICS

ANSWER KEY

QUESTIONS FOR SHORT ANSWER


 
1. Magnitude | a| will remain uncharged. B = a

 
∆a = B − A = a 2 + a 2 + 2a 2 cos( π − dθ)

–a
= 2a2 (1 − cos θ) ⇒ 2a2 (1 − 1 + 2 sin 2dθ / 2) = 2a sin d θ/2

2 Speedometer measure speed of car as it only gives the magnitude.


3 When particle is moving with constant velocity its average velocity and instantaneous velocity will be
same and magnitude of instantaneous velocity will also be same.

 ∆S 
4 VAvg = , ∆S = VAvg × ∆t
∆t

 ∆S   x
5 VAvg = ∆ S = 0 with zero displacement non zero VAvg is not
∆t

possible zero displacement and non zero V is possible if particle
t
is reversing and coming to starting point. Show on x-t graph by an example.

6 Speed of projectile is smallest at the highest point.


7 Both the ball will hit the ground with same speed.
8 If sack of rice is dropped when it is just above the centre it will fall ahead of circle because sack will
have velocity same as plane in horizontal direction.
9 Ist Curve : at particular time x has more than one value hence not a 1-D motion.

IInd Curve : | V| cannot be negative

IIIrd Curve : Length of a moving body can not decrease with time
10 Ist Curve : A ball moving forward collides with surface rebounds and stops after IInd collision
IInd Curve : A ball repeatedly making inelastic collisions with floor.
IIIrd Curve : Collision of a ball with surface. {Surface has large velocity for short time}

11 (a) is incorrect car can not travel around track with constant velocity as direction is continuously
changing.
(b) correct

12 Ball at maximum height V = 0 for just an instant but acceleration due to gravity.
1
13 Vf = 2gH . Let balls meet after t sec. h1
1 2 1 2 H X
h1 = gt and h2 = Vf t = gt
2 2 h2
H V0=Vf
h1 + h2 = H = Vf t H = 2 gH t t = 2
2g
1 H H
∴ h1 = g = hence they will meet above half height of building.
2 2g 4

394,50 - Rajeev Gandhi Nagar Kota, Ph. No. : 93141-87482, 0744-2209671


IVRS No : 0744-2439051, 52, 53, www. motioniitjee.com , hr@motioniitjee.com

Downloaded from www.iitjeephysics4u.com


KINEMATICS Page # 95

14 Initially bullet is at rest u = 0 V2 = 0 + 2as

V2
∴ a= muzzle velocity is more for short barrl and S is also less hence acceleration will be more in that case.
2S

15 Hence we can not conclude that velocity of boat is 5 m/sec w.r.t. shore

VBottle = Vriver ; VB – VR = 5
16 Yes wrench will hit at the same place on the deck irrespective of that boat is at rest or moving because
when boat is at rest wrench will have zero horizontally velocity and when boat is moving both will have
same horizontal velocity.
17 Acceleration of the projectile remains constant throughout the journey = g
18 (a) In child point of view range will be same in both the cases.
(b) In ground frame of reference
VCT = VC – VT
VC = VCT + VT
For front range Vcannon = VC cos θ + VT Range will be more
For Rear range Vcannon = VC cos θ – VT Range will be less

d
19 d t= for tmin cos θ = 1 maximum Hence A will reach opposite end in least time
Vbr Vbr cos θ

ANSWER KEY EXERCISE - I

1. B 2. B 3. A 4. B 5. A 6. D 7. C 8. B 9. B 10. D 11. C 12. B 13. B 14. A

15. D 16. C 17. B 18. C 19. C 20 D 21. C 22. D 23. C 24. B 25. B

26. D 27. C 28. B 29. C 30. A 31. A 32. A 33. C 34. A 35. D 36. D

37. C 38. C 39. B 40. B 41. D 42. D 43. B 44. A 45. D 46. A 47. B

48. C 49. B 50. C 51. D 52. B 53. B 54. A 55. C 56. C 57. C 58. C

59. A 60. A 61. C 62. C 63. D 64. B 65. C 66. D 67. B 68. D 69. D

70. C 71. A 72. A 73. A 74. A 75. B 76. B 77. C 78. B 79. B

ANSWER KEY EXERCISE - II


1. A,B,C,D 2. A,C 3. B,C,D 4. B,D 5. C,D 6. A,C 7. A,C 8. A,D 9. C

10. A,B,C,D 11. B,C,D 12. A,C,D 13. A,B,C 14. C,D 15. D,C 16. A,B,C,D 17. A,B 18. A,B,C,D
19. A,C,D 20. B

394,50 - Rajeev Gandhi Nagar Kota, Ph. No. : 93141-87482, 0744-2209671


IVRS No : 0744-2439051, 52, 53, www. motioniitjee.com , hr@motioniitjee.com

Downloaded from www.iitjeephysics4u.com


Page # 96 KINEMATICS

ANSWER KEY EXERCISE - III

1. (a) y2 + 8y + 12 = x ; (b) crosses x axis when t = 4 sec, crosses y axis when t = ± 2 sec.

2. (a) 5i + 3j + 8k, (b) 2i + 16k, (c) 7 2 , (d) 2 65 3. –18 tj – 3 cos t k ; 3π ; 3 4 + π 4

1 3  
4. (a) m / s , (b) m / s 2 , (c) 5. v = –30 i – 40 j, a = –16 i – 8 j 6. a/2b, a2/4b
3 2

2 38 v0 a vel π 2
7. (a) m , (b) m 8. 42 km/hr 9. 10.  3  cm/min 11. 25 m
3 3 – +  

 h
12. (a) 2.7 km; (b) 60 m/s; (c) 225 m and 2.25 km 13. 240 m 14.   15. 50 m
 8g 

16. 100 m, zero 17. 5 s 18. 36.2 sec. 19. 20 5 20. 20 sec 21. 60, 2 m/sec.

22. u = 50 ( 3 – 1) m/sec., H = 125 (– 3 + 2)m 23. (i) 1503.2 m (ii) 35.54 sec (iii) 3970.56 m

24. 100/3 m/s 25. 10 m/s 26. 75 m 27. R + 2H 28. 6 m/s 29. 20 × 2/3

–1  1  4
30. 10 m 31. 50 3 cm 32. tan–1 (1/2) 33. tan–1(3) 34. θ = tan   ,
2 3

35. 200 m, 20 m/min, 12 m/min

ANSWER KEY EXERCISE - IV


πv 4
1. 122.7 km/hr 2.
g2
3. 60°, tan
–1
( 3/2 ) 4. u = 3m/s 5. (a) 10 ms–1,(b) 2 sec, (c)5 m, (d)

1
16.25 m, (e) 20 m 6. sec 7. 5 m/sec 8. 1 sec 9. 2 tan–1 (1/3) 10. θ = 37°, v = 6 m/s
3

ANSWER KEY EXERCISE - V

a2 u 2 sin 2α u cos(α + θ)
1. (a) 1 sec, (b) ( (5 3 m, 5 m) 2. , tan –1 a 3. (a) , (b) v =
4b g cos θ cos θ

4. B 5. A 6. u = 7.29 m/s, t = 1 sec 7. (a) 45°, (b) 2m/sec

8. B 9. C 10. B 11. B 12. 5 m/s2

394,50 - Rajeev Gandhi Nagar Kota, Ph. No. : 93141-87482, 0744-2209671


IVRS No : 0744-2439051, 52, 53, www. motioniitjee.com , hr@motioniitjee.com

Downloaded from www.iitjeephysics4u.com

Вам также может понравиться